4/28/2005

Friedman's Suggestions (from mit.bbs)

PRINCIPLES OF ECONOMICS

Contents

Have An Agenda, and Know Why It's Important

Be Awake; Look Around

Be Ambitious, But Not Too Ambitions

Have Staying Power

Decide Who Is the Audience, and Learn How to Reach It

Keep Things in Perspective

Most kinds of intellectual endeavor hold out the prospect of a particular satisfaction, that associated with expanding the possibilities for thinking about ourselves and the world in which we live. Economics is no exception. To be sure, economics does have its particularities--an idiosyncratic mixture of a priori theorizing and data-based empiricism, a commitment to apply the scientific method despite the inability to carry out replicable or even controlled experiments, a closeness to certain contentious political issues, and so on--and as economists we are rightly aware of them. But in the end it is the similarity to other avenues of the intellectual enterprise that is more compelling, including not just the physical sciences but history, philosophy, and even literature and the arts. As a consequence, the core principles of what makes for good economics are probably pretty similar to the route to finding satisfaction in most other intellectual pursuits: Have an agenda, and know why it's important. Be awake; look around. Be ambitious but not over-ambitious. Have staying power. Decide who is the audience, and learn how to reach it. Keep things in perspective.

These principles may sound obvious, or empty, or both, but I doubt that when I first became an economist I understood them in the way I do now, and I certainly don't pretend that I have unfailingly adhered to them at every point since. Economics, again in common with so many other endeavors, is very much a matter of learning by doing. I think I have learned, along the way, about what the satisfactions of doing economics are and what general working principles are helpful for achieving them. My object here is therefore not so much to report what I have done, or even what I always now do, but to extract from both what I believe works best.

Have An Agenda, and Know Why It's Important

The agenda of economics is to understand an important aspect of the human experience: why we behave as we do in certain contexts, both individually and collectively; what consequences follow from the fact that we behave in this way; and in light of this behavior and its predictable consequences, what we might do, either individually or collectively, to improve our lot in this world. Saying this, especially to trained professionals, may seem either trivial or trite. But it is surely not trivial, and if it is trite it is also very often forgotten.

A distinction between empirical and axiomatic approaches to the questions at hand is familiar in many sciences, and economics is again no exception. In my own work I have always felt more comfortable following an empirical approach, by which I mean starting with some aspect of economic behavior that we actually observe and seeking an explanation. Why do aggregate production and income grow faster at some times than others, and sometimes not at all? Why do interest rates vary, and why do they covary among one another, as they do? How do businesses decide how much to borrow, and in what form? The axiomatic approach, starting with a few first principles and logically determining what consequences follow from specific additional assumptions, has been just as central to economic inquiry if not more so. But the greater risk, I usually think, is that of applying impeccable logic to proceed from assumption to conclusion when neither bears much actual connection to the behavior of the real people and institutions, and hence the real economies, that I regard as our subject's proper object of study.

Under either approach, however, it is essential to be able to say why the effort is worth while in the first place. The initial question I try to answer for myself whenever I embark on a fresh project--when I begin a new (at least for me) line of research, or pursue an intriguing loose end left by work I have been doing, or offer a new course for students--is "Why am I doing this?" What can I learn, and why might that be valuable? Is the behavior I want to examine important in its own right? Or is the knowledge to be gained important because it might shed light on some related question? In that case, why is this other question important? The main reason I find it more comfortable to begin work from an empirical direction is that that way I find it easier to answer these questions--and they can often be hard questions--about what I am doing and why.

By contrast, setting out to do research without thinking through why it is otentially worth doing is like trying archery in the dark. There is some small probability that any randomly directed arrow will reach the target, and with enough bowmen taking enough blind shots, inevitably some will. Similarly, some few economists who are entirely unaware of the broader context that might make others value their findings will probably hit the bull's eye anyway. But the likelihood of doing so is far greater if a keen sensitivity to just that broader context shapes not only the selection of the question to be attacked but also the means of investigating it. Some empirical findings, and some theorems, become important because they give answers to questions people genuinely want answered; others don't because they don't.

The immediate implication of this seemingly obvious point is that, with limited time to spend, not everything that is doable is worth doing. Specifically, not every extension to a theorem is worth proving, nor is every empirical observation worth explaining. Even more to the point, especially for purposes of younger researchers, the mere fact that so-and-so has published a paper on some subject or other does not by itself make that subject worth further investigation. (It may not have merited so-and-so's original paper either, but that's a different matter.) Reading the journals is an excellent way to learn research methods; it's a poor way to choose research topics.

What does lead to good research questions? Here too, I have usually found the attractions of the empirical approach compelling. If the object of economics in the first place is to understand certain aspects of behavior by individuals and institutions, or its consequences for whole economies, then the most straightforward way to find simulating topics is to observe that behavior. For behavior in the aggregate, that mostly means listening to the questions concerned people are asking. For individual behavior, just watch. For behavior by institutions, find a way to watch.

When I was a graduate student, I took on a series of either part-time or limited-term assignments for various components of the Federal Reserve System. One was to study the conceptual structure underlying the Board staff's presentation of information to the Federal Open Market Committee. (The key question was how to structure the conditionality of future economic outcomes on the Committee's own monetary policy decisions.) Another was to serve on a committee. made up of representatives from the Board and some of the regional Banks, to recommend how best to introduce money growth targets into the Open Market Committee's policy decisions. (In those days--as is the case again today--the Federal Reserve didn't use money growth targets.) The first job I took after finishing my formal education was working at a New York investment banking firm. I wasn't in the economics department (the firm didn't have one at the time) but rather divided my time between the part of the business that worked with corporate clients on their bond issues and the part that sold the securities to institutional investors. Much of my subsequent research--on the theory of economic policy, on targets and instruments of monetary policy, on corporate borrowing decisions, on portfolio behavior and the determination of interest rates, on the role of credit markets in influencing macroeconomic activity--grew out of these early first-hand exposures to actual economic behavior. For just the same reason, in more recent years I have valued highly the opportunity to work with some financial institutions on the kind of sustained basis over time that has let me watch, and ask questions about, how they conduct their business. (By contrast, I rarely accept one-shot assignments.)

Regardless of whether research is empirical or axiomatic, however, the question of importance remains essential. The value to me of those early opportunities to see some interesting institutions at close hand was not just in suggesting research questions but in showing me who wanted to know the answers to what questions, and why. The object--the source of satisfaction from the enterprise as a whole--is not to maximize the number of published papers to one's credit but to shed as much useful light on the subject as possible. If the first question I ask myself is why I think a potential topic is important, the second is who will be interested, or better yet surprised--or even better still, discoretired--by the potential findings. In much of my work on monetary policy, for example, the objective has been to show that key aspects of behavior in the economy in which we happen to live make mechanistic rules for central bank conduct unhelpful. I thought that work was worth doing (and I still think so) not just because the subject is inherently important but also because so many people prefer to think the opposite. The ultimate question for any researcher is always how people will see that particular slice of the subject differently because he or she has worked on it.

Be Awake; Look Around

If the objective is to shape one's own research agenda in light of the actual behavior we observe and seek to explain, it helps to pay attention. New phenomena--the corporate debt explosion of the 1980s, for example, or the OPEC oil price increases in the 1970s--are especially interesting, either because they represent a new form of behavior or because they provide a new window for analyzing aspects of economic behavior that are already familiar but only from other lights. But it is striking how much there is to learn from simply watching people and institutions do what they have always done, or from listening to people describe what they do.

One reason this kind of observation of the ordinary is so important is that economic thinking (that is, the thinking of professional economists) is so often blinkered by the assumptions we impose and, moreover, that those assumptions are themselves so arbitrary. Aspects of everyday behavior that do not fit conveniently within the framework determined by whatever assumptions are fashionable at the moment remain, for all practical purposes, invisible. An example on which I worked for a while, alas in the days before doing so was fully respectable, is credit rationing. It is embarrassing today to recall the air of derisive ridicule with which distinguished economists not long ago dismissed even the possibility that lenders might adopt any strategy other than raising the interest rates they charged so as to bring loan demand into equality with loan supply. The fact that almost everybody who knew at close hand about loan markets thought bankers did sometimes ration credit, and said so, was simply no match for the fact that there was no formal maximizing model capable of rationalizing such behavior. But as soon as someone thought to bring to bear in a formal way such notions as asymmetric information, adverse selection and moral hazard, then of course credit rationing might occur, and a subject once better ignored in polite professional company became open game for accepted scientific investigation.

The point is not that simplifying assumptions (in this case, perfect information) are not useful--indeed, they are necessary to carry out any serious analysis--but that the conventionally accepted simplifying assumptions of the day are often highly arbitrary and hence subject to change, and therefore that there is no shame in choosing new ones when observed behavior doesn't fit snugly within the usuals. Just as for a long time the prevailing theoretically correct thinking rejected even the possibility of credit rationing, for a time (mercifully brief) the prevailing theoretically correct opinion took on faith that because people's expectations were rational, pre-announced monetary policy actions simply couldn't affect output or employment. In this case it wasn't long before numerous economists pointed out that the models that gave rise to this conclusion rested not only on a quite specific (and, on reflection, perhaps unsuitable) notion of "rationality" but also on a host of other questionable assumptions like frictionless adjustment of prices and wages. Even so, for some years every conference on macroeconomics was forced to listen to the repeated assertion that economists would have to proceed as if this model were a good characterization of the world because "it's the only well worked out model we've got." Here again, the presumption was that behavior simply could not exist because there was (as yet) no maximizing model to account for it.

For purposes of doing theoretical economics, the antidote to such wrong-headedness is to look for new assumptions. As in the credit rationing example, maybe information isn't perfect. As in the monetary policy example, maybe markets don't adjust frictionlessly. The range of conventional assumptions subject to challenge is enormous. Maybe personal utilities aren't independent. Maybe aggregation does matter. Maybe the dependence of this on that isn't linear. (Much interesting literature in recent years has usefully explored conditions that give rise to multiple equilibria, but of course that possibility follows immediately when the relevant behavioral relationships are nonlinear.)

For purposes of empirical work, the message is that an observed phenomenon is no less interesting to study just because nobody has written down a maximizing model to explain it. Indeed, in that case empirical findings may be the best clue to what assumptions need changing in order to deliver just such a model. As I have listened over time to the questions that my friends in public policy institutions and in private business firms ask, I am often struck by how little we--economists--have to say about what they want to know. (Sometimes I am struck by how much we know, but here my point is different.) In part, these lacunae persist because it is genuinely hard to learn about some kinds of behavior and their consequences. But in some cases we have just not asked the right questions.

A whole other reason for paying attention to what is happening, and to what people are saying, is that the behavior we study changes. Not behavior in the sense of the ultimate underlying "meta-model," of course; but what economists actually study is not the meta-model but behavior in one usually tiny piece of it that takes the rest as given. For just this reason, institutions--legal arrangements, business practices, social mores, and so on--matter importantly for many aspects of economic behavior. And when those institutions change, economic relationships that depend on them, in ways either obvious or subtle, change as well. There is a tautological sense in which it must be true that inflation is "always and everywhere a monetary phenomenon," but that is not the sense in which many people in the United States understood this notion a couple of decades ago, before observed inflation and the conventional M's began to go their separate ways. Simply to assume that answers to important questions derived from past experience remain right answers is to miss much of what is interesting and important about our subject.

Finally, yet another reason why it helps to look around is that the questions people ask change too. To be sure, issues like the real costs of disinflation, or the value of creating a market for price-indexed securities, or the gain in efficiency from indexing the tax code, are always valid subjects for economic research. But it is hardly surprising that more people want to pay attention to the findings of research on those questions when prices are rising rapidly than when prices are more nearly stable. For the same reason, whether government budget deficits in a fully employed economy crowd out private capital formation, or under what circumstances a deficit would have to be monetized, was not much of an issue in the United States before the 1980s. This did not mean that there was no point in addressing such questions before then. But the context that determines whether any specific piece of research speaks to a matter of broad concern, and hence has the potential ability to have significant impact on widespread thinking, clearly changed. People who don't look around don't notice.

Be Ambitious, But Not Too Ambitions

Rabbi Tarphon, a noted sage of the first century, declared that "You are not required to finish the task, but neither are you free to neglect it altogether." Tarphon's injunction has always seemed to me a useful beacon for researchers, especially in economics. The part about not neglecting the task is obvious enough, but I think the idea that finishing it is not required is useful, indeed important, for maintaining a sense of purpose.

A curious outsider, taking a fresh look at economics, is less likely to be struck by how much we know than how much we don't. Few established empirical findings are genuinely stable across time and space. Most theoretical results depend on a vast array of simplifying assumptions. Many of these assumptions--atomistic competitors, independent utilities, linear functional relationships, identical "representative" agents, and so on--have over time become sufficiently conventional in the eyes of practicing researchers that they seem to require no justification (indeed, they are often taken for granted without even an explicit mention); but to the thoughtful outsider they may seem not just strange but factually wrong (as, of course, they are). Especially for someone newly beginning a research career, the resulting temptation can be to reject the entire working apparatus of modern economics as epistemologically flawed, and set out to erect a whole new edifice in its place.

That strategy is a recipe for failure. Discontent with the artificiality of whatever set of arbitrary assumptions is in fashion at the moment is a healthy motivation for making progress. Seeking to abandon useful workaday assumptions whosesale is a bar to making any progress whatever. There is tension but not conflict in wanting to change many aspects of how economists think yet actually investigating only one such change at a time. There is conflict but not fundamental inconsistency in attacking one unappealing assumption in one line of research while going ahead to use that same assumption, unappealing though it may be, in another line of research where the focus is different. The history of our subject shows that progress comes incrementally, in the middle ground between finishing the task and neglecting it altogether. Economics is a task that no one is required to finish, not even in one lifetime much less in one paper. The practical consequences of trying to finish this particular task are often indistinguishable from those of simply neglecting it.

A different form of over-ambition in economic research is the Icarus problem: trying to fly too close to the universal sun, in the sense of supposing that a particular piece of research comes closer to the ultimate meta-model than it (or anything else that is really feasible) can. The meta-model by definition takes all factors into account. It doesn't change with circumstances not controlled for, because it controls for all relevant circumstances. By contrast, fruitful economic research focuses on only a few key variables at a time, leaving the rest aside. This is not a flaw to be endlessly lamented but a fact to be usefully remembered.

In particular, this means that the universality to which we might like to pretend for our findings, because we appropriately aspire to it, just isn't there. Our results are local results. As environments and institutions change, so will even our favorite empirical relationships, and even our favorite theorems depend on more assumptions than we usually enumerate. This does not make our work valueless, just limited. By now many of the empirical relationships describing credit market behavior (and especially the borrowing behavior of firms) that I labored to investigate some years ago no longer correspond to current data. I may be sorry about that, but l do not have to regard the basic lessons of that work as worthless. The models I used were at best only small pieces of the meta-model, and as factors that I omitted from my analysis changed, so did the observed behavior.

A closely related temptation, also to be avoided, is the monocular syndrome--that is, the tendency of economists to assert monocausal explanations for complex phenomena. For many if not most problems, the most effective research strategy is not only to work on explaining one aspect of economic behavior at a time but also to focus on only one part of the explanation at a time. Not infrequently, a useful exercise is even to see how far it is possible to go in explaining the behavior in question on the basis of the one causal factor under investigation at the moment. All this makes for good economics. But it is important not to take such exercises too seriously, and so conclude that some important aspect of economic behavior really does have only one causal force behind it.

For reasons that are closely related to both the Icarus problem and the monocular syndrome, I have always been reluctant to extrapolate what we know from one context to others where essential aspects of the environment are different. A useful example is the study of hyperinflation (about which I too once wrote a paper). Hyperinflations are certainly interesting phenomena in their own right, not least because of their sometimes powerful political consequences. But can we apply the lessons drawn from examining the demand for money during hyperinflations, when one influence on portfolio choice is enlarged to a magnitude such that it actually does dwarf all others, to draw inferences about money demand under more ordinary circumstances? Can the experience of ending hyperinflations usefully inform our estimate of the likely costs of a transition from moderate but persistent inflation to price stability? I am usually inclined to be skeptical of such extrapolations. Instead, if I want to learn about a question, I try to study it in its own context. (For just the same reason, I almost always disappoint foreign journalists who ask me what advice I would give their own governments. I'm not being either politically careful or overly polite; I just don't think I know.)

Yet a different form of over-ambition in economic research is to require too much of a model, and in particular to strive for false depth. Here the example that comes most readily to mind is the treatment of the demand for money. Some years ago it became fashionable to argue that it is illegitimate to draw inferences about monetary policy from any model that lacks an internal explanation for why people hold money. (For reasons that I never understood, in much of this literature it was further regarded as bad form to acknowledge that the reason for holding money might have something to do with its usefulness in effecting transactions.) Why people hold money is surely a useful and important question for economic research to address. But it is also surely useful to do different research on the basis of assuming that people in fact do hold money and proceeding on from there. Insisting that both efforts must cohabit within the same model is a bit like wanting the driver's manual to contain a chapter on the origins of the convention that cars go on green and stop on red, or on why different countries opt for the right or the left side of the road. Division of labor does have its uses.

Have Staying Power

One of the hardest things to decide in pursing any agenda, including an intellectual one, is how long to stay the course. Nobody wants to give up too easily, just because people are initially resistant to a seemingly worthwhile idea, or because a few pieces of partial evidence point the other way. At the same time, nobody wants to hold onto an idea long after overwhelming evidence has contradicted it. Resolving this tension is rarely easy.

On balance, though, I'm usually inclined to stay the course more persistently than not. One reason is that much of economics suffers deeply from the short sample problem. It is not just that we can't conduct replicated experiments to address most economic questions, or that the one history we have does not represent a controlled experiment. The added difficulty is that for purposes of many of the questions we want to ask, that history is short. It is short in part because environments and institutions matter, and they change. We may have data on the volume of bank loans extending back into the nineteenth century, but the loan market today differs from the markets of earlier eras in so many ways--loan securitization, hedging capabilities, and competition from the commercial paper market as well as from abroad come immediately to mind--that the relevance of data from decades ago is of limited value for many research purposes. Our one history is short also because observations are not independent across either time or space. Regardless of whether we divide the data yearly, quarterly or monthly, how many genuinely independent observations does the post-war rise and then decline of inflation contain? How many independent observations does the growth experience of twenty-four OECD countries contain? While this line of thinking is certainly not ground for despairing of ever learning from empirical analysis, it does make me pause before too quickly changing my mind because I have seen one new set of regressions.

The continually shifting tide of fashion in acceptable assumptions provides yet another reason for resisting pressure to abandon an idea that usefully seems to explain the behavior we observe. As the example of credit rationing shows, what respectable opinion deems impossible can become part of what "everybody knows" with astonishing suddenness. I sometimes wonder whether I should have continued doing research on credit rationing, since I have always believed it is an important aspect of bank behavior. I know I would not have worked out the crucial maximizing model based on asymmetric information and adverse selection--my personal toolkit is not well designed for that particular task--but I am at least curious about what evidence and insights a sustained program of empirical research on just this aspect of financial behavior might have produced.

But saying that one should stay the course despite opposition and even some contrary evidence is not to say never to change one's mind. The object, after all, is to learn. Sometimes observed behavior actually does present pretty dramatic statements one way or the other. For example, I used to be receptive to the idea that saving is positively interest elastic, and I therefore was sympathetic to the general class of policy proposals for stimulating private saving to which a positive elasticity gives rise. After the decline in U.S. saving rates in the 1980s, in the face of truly extraordinary increases in real after-tax returns, I have changed my view. (I think the same decline in saving, in the face of record government deficits at full employment, was likewise pretty devastating to the notion of Ricardian equivalence; but on that one I was a disbeliever much earlier on.)

I have also learned over time that the United States is much more of an open economy that I used to think. The biggest mistake I made in thinking about the policy issues of the last decade and a half was to under-estimate how much the U.S. Government's budget deficit would affect the country's net export balance (and thereby change the direction of capital flows), and correspondingly over-estimate how much it would effect our domestic investment. The standard closed-economy model that shapes my most basic economic intuitions just wasn't adequate. I've also learned over time that price inflation is a much more serious problem than I used to believe--even though I still don't think our profession (me included) has much understanding of why.

So, changing one's mind is important too. But on balance, when the issue is in doubt, I'm inclined to stay the course and wait for others to change theirs. Most of the pictures on the walls in my study are portraits. The largest by far is of Winston Churchill, a man of determinedly held views if there ever was one. From the late 1920s on, Churchill was not just out of office but without real influence, his views rejected and ultimately ridiculed by the conventional wisdom of the time. He did not hold public office again until the fact of the opening of the war made it obvious that he had been right all along, and he became prime minister just nine months later. He was then sixty-five years old.

Decide Who Is the Audience, and Learn How to Reach It

I occasionally hear it said of some economist or other that he would be happiest just writing papers and putting them in his desk drawer, deriving ample satisfaction from the repeated act of analytical creation without ever showing its fruits to other people. I have never met such an economist. In a very few instances I have heard an economist I knew described in this way, but in each case I knew the person well enough to realize that what was said about him wasn't true.

Most economists, perhaps all of us, want not only to do interesting thinking but to communicate it to others. More than that, most of us want to persuade other people to accept our thinking. The principal means of communication are talking and writing. Of the two, writing is what lasts.

In our era writing by academics in general, and by economists in particular, has become the standard butt of stock jokes. I think that's unfair. To be sure, much writing by economists is simply bad. But much is quite good, and many economists write extremely well. Making younger economists think that they have somehow inherited a generic professional disability, a kind of congenital handicap against which they will have to contend for the entirety of their careers, does no one a service. The point is simply that writing well is an important part of communicating effectively, and an especially important part of persuading effectively, and that this is true for economists in the same way it is true for people who seek to communicate and persuade in countless other professions. As with anything else, the main secret to success is working hard at it. In the case of writing, this mostly means going back to it again and again and again--to find just the right word, to restructure a sentence or a paragraph, to insert a new thought, and sometimes even to change around the whole logical flow. My colleague John Kenneth Galbraith once referred to "the appearance of effortless ease that creeps into my (Ken's) prose on about the eighth draft." He was indirectly offering me advice, and I've tried to take it seriously.

Some dimensions of the matter, however, probably are harder for economists. The one I think is especially important is that many economists want-- appropriately-- to communicate with several different audiences who happen to use different languages. We want, in the first instance, to speak among ourselves. But academic economists also need to speak to their students, and business economists need to speak to others in their firm or to their customers. Many economists also want to speak to policy makers from time to time. Some occasionally want to address a more general public.

The problem of different languages is real. My first exposure to the Federal Reserve System was a summer job in the research department of the Federal Reserve Bank of New York. By then I had studied economics for four years in college and two more in graduate school. Although most of the people I talked with at the bank that summer were professional economists, I quickly realized that I just didn't understand what they were saying. (I don't mean that I didn't understand why the theory underlying what they said was valid; I literally did not understand many of the conversations taking place.) As I eventually discovered, they were in fact talking about things I had learned about. But they used a different vocabulary than I knew, and they left much of the context implicit.

Vocabulary and context are crucial to communicating effectively, and it makes little sense to address an audience in anything other than its own vocabulary or without providing the right context. I think much of the usual popular derision of academic writing stems from the reaction of one audience, either practitioners or perhaps even interested laymen, to material written for research professionals who constitute a wholly different audience. The vocabulary is strange, and even the words that should be familiar lack the context to give them genuine meaning.

American populism has always exhibited an antiIntellectual strain, and so the Congressman who wants to score points by making fun of the silly professors can easily draw laughs by reading selected passages from the professional journals in just about any academic discipline. While few layman are inclined to think they should be able to understand astrophysics or Byzantine theology, however, many non-economists do think they should be able to understand matters of economics. More importantly, citizens in a democratic republic have not only a right but, indeed, an obligation to understand major issues of economic policy. While I am often struck by how little economists know about the questions that interested laymen or public policy officials or business executives ask, in many cases I think we do know much that is useful. But it remains to communicate what we know to them. I think it is to our credit that so many economists want to address these nonprofessional audiences. But we can do so effectively only if we use a vocabulary that they can understand and if we provide the context that makes what we say meaningful.

Here too, what makes this kind of communication succeed is largely putting effort into it. If I think Congressmen, or bankers, or businessmen may be interested in the findings of the research I have been doing, I have to accept the fact that simply sending around reprints of my latest journal articles won't do. I have to decide whether I want to convey my ideas to those audiences or not. And if I do, then I know I have to write an account of those ideas directed at the audience I want to reach.

Some of my academic colleagues who read my Day of Reckoning book, as well as some friends in the financial community, told me they would have found the book easier to follow--not mention a lot shorter--if I had included some tables and time series plots to exhibit the most important trends and relationships in the data. They were right. (One person, whom I didn't know, sent me a letter saying he assumed I must have been writing from a set of tables, and asking if I could provide him with a copy.) But I didn't write that particular book for them. I deliberately chose a purely literary presentation--no tables, no data plots, no diagrams, and certainly no equations--because I wanted people to read it who would simply have put it down if they had paged through it and spotted any of these devices. I knew that once people actually decided to read the book, some well chosen tables and plots would have made it easier for many if not most. But I decided that for this particular effort at communication, the audience I wanted to reach included large numbers of people who, if they saw tables and data plots, would probably never read it at all.

Writing a book this way--producing a purely literary presentation of a subject we economists usually discuss among ourselves using both short-hand and short cuts--was, of course, time consuming. It took away from research I otherwise could have done. (That book was not research; I like to think of it as high-class journalism.) But I took the time because I thought that that particular effort at communicating, and persuading, was important. I felt about it, in some ways, a sense of moral obligation.

Keep Things in Perspective

One of our Presidents once remarked that a major personal challenge for people charged with public responsibility, especially at high levels, is to take their decisions appropriately seriously yet not take themselves too seriously. I think scholars face the same tension. We devote our lives to research and teaching on issues that we deem important. We take these issues and our work on them very seriously, and we are right to do so. But we do ourselves--and others too--a disservice if we fall into the trap of also taking ourselves too seriously.

Steering clear of this particular temptation is no doubt a matter of many dimensions, but in my own experience two especially stand out. First, some of the friendships I have valued most over the years have been (and still are) with economists whose views often directly contradict my own. We disagree with each other in our papers, we debate each other at conferences, and we argue with each other when we get together just to enjoy each other's company. I admire these friends, and I have learned from them. But more important, in the end, they are my friends and I value them simply for that. Another eminent sage, Isi ben Judah, asked "Why do scholars die prematurely?" His answer? "Because they abuse one another." Taking ourselves less seriously than we take the ideas on which we work may or may not enable us to live longer, but I think it does help to keep our work from obstructing personal relationships that can be deeply satisfying.

The other sense in which trying not to take ourselves too seriously has been important to me reflects a lesson I learned in a vivid way years ago when I worked in investment banking. I not infrequently worked on assignments with Robert Baldwin, a quite senior partner who soon afterward became head of the firm. I remember especially clearly the experience, on several occasions, of sitting in his office with a team of other partners and staff members, trying to schedule an important meeting with one major client or other. Somebody would suggest a date, everybody in the room would agree, and then Bob would check his calendar and declare that that was impossible because it was the day of his son's school play (or hockey game, or whatever was the particular event that time). Everybody else would exchange knowing glances, as if to say "This guy is nuts but we have to humor him," and eventually somebody would go on to suggest a new date. In the meanwhile, my own (silent) reaction was more along the lines of "This guy is the only one here who understands what's important."

Balancing our personal and our professional involvements is a tension that we all face. As is usually the case with such tensions, having a clear sense of priorities helps. I've always had mine pretty clear. My wife and sons come first.

But all this brings me back to where I began: Having an agenda is crucial. So is knowing why it's important.

~~~~~~~~

By Benjamin M. Friedman

William Joseph Maier Professor of Political Economy, Harvard University. This paper will appear in Michael Szenberg, ed. Passion and Craft, Economists at Work. Ann Arbor: Michigan University Press

Read more!

Mankiw's research experience (from mit.bbs)

MY RULES OF THUMB

Contents Rule No. 1: Learn from the Right Mentors Rule No. 2: Work With Good Co-authors Rule No. 3: Have Broad Interests Rule No. 4: Allocate Time with Care Rule No. 5: Write Well Rule No. 6: Have Fun

My assignment is to describe how I work. I take on this task with mixed feelings. One can easily become vain in the process of public introspection, and vanity is a trait best left private. It is not entirely clear to me why anyone should care about my idiosyncrasies--except, perhaps, for my colleagues, students, and family, who have no choice but to live with them.

Yet when other economists write essays of this sort, I enjoy reading them. I like to think that these essays edify me in some way, but at the very least they appeal to the voyeur in me. So, I figured, others may learn from a brief essay about how I work. Or, at least, they may be amused by it.

I have organized this essay around six rules of thumb that I follow as I go about my working life. I have chosen these rules largely for their positive value--they describe my behavior. I do not pretend that the way I work necessarily holds any prescriptive value for anyone else. But it may. If these rules of thumb ring true to others and help them to run their lives, so much the better.

Rule No. 1: Learn from the Right Mentors I learned how to practice my trade from four distinguished economists. Perhaps the reason was good career planning on my part. More likely, it was just good luck.

In the spring of 1977, as a freshman at Princeton, I took Principles of Microeconomics from Harvey Rosen. Harvey was an excellent teacher. I remember finding the material easy and, at the same time, feeling that I was learning a tremendous amount. Each lecture was filled with insights that were novel, profound, and so stunningly obvious that it seemed I should have known them all my life. But, of course, I didn't. Principles of microeconomics was the most eye-opening course I have ever taken. All subsequent courses in economics have exhibited the property of diminishing returns.

For reasons that are a mystery to me now, Harvey hired me as a research assistant for the summer after my freshman year. I knew very little economics, for I had taken only the two principles courses. I did know something about computer programming (a fact that surprises my own research assistants, for changes in technology have made this human capital long obsolete). For whatever reason, Harvey did hire me, and the experience proved invaluable. I knew so little that Harvey had to teach me whatever he needed me to know. Spending a summer being tutored by a top teacher and scholar is the best learning experience I can imagine. To this day, I have never learned so much in so short a period of time.

Eventually, my interests drifted toward macroeconomics. As a senior at Princeton, I took graduate macroeconomics from Alan Blinder, another excellent teacher. At the same time, I wrote my senior thesis under Alan's supervision. In the thesis, I tried to make sense of the cyclical behavior of the real wage, which has puzzled macroeconomists at least since the publication of Keynes's General Theory. Part of my senior thesis became a paper co-authored with Alan, which we later published in the Journal of Monetary Economics. More important, as I worked on the thesis, I became convinced that imperfections in goods markets were at least as important as imperfections in labor markets for understanding the business cycle. This conviction eventually led to my involvement in a line of research now called New Keynesian Economics.

When I entered MIT's graduate program in the fall of 1980, Larry Summers was a young assistant professor. Larry's enthusiasm, breadth of knowledge, and quick mind attracted me, and we spoke together at MIT during the year and at the NBER during the following summer. When Martin Feldstein brought Larry to work at the Council of Economic Advisers in September 1982, Larry brought me along with him. I was fortunate to be able to work closely with Larry during the brief period when he was already a great economist but not yet a famous one.

When I returned to MIT, Stanley Fischer served as my dissertation adviser, as he did for a remarkable number of students in my class. Stan was a model of professorial balance. As a lecturer, he gave clear and even-handed presentations in a field that can be confusing and divisive. As an adviser, he encouraged students to pursue their interests with the highest standards of rigor without imposing his own intellectual agenda on them. My dissertation, like most in recent years, was a collection of loosely related papers bound together for the sole purpose of getting a degree. It bore the soporific title "Essays on Consumption."

When I look back at these four mentors--Rosen, Blinder, Summers, and Fischer--I see in them various characteristics that I have developed over time. They are prolific writers. Their research tends to be empirical and policy-oriented. They take teaching seriously.

All of my mentors have shown interest in reaching a broader audience than can be found writing in academic journals. All four of them have taken time away from academia to work in policy jobs in Washington. Three out of four have written textbooks, and two of them have written more than one textbook.

It is easy to see why mentors matter. Mentors determine your professional outlook in much the way that parents determine your personal outlook. Mentors, like parents, give you your values. They teach you what kind of behavior to respect and what kind to avoid. And they teach these lessons indirectly, more often through their actions than through their words.

The major difference is that your parents are predetermined. You get to choose your mentors.

Rule No. 2: Work With Good Co-authors I have been lucky to be able to work with many talented co-authors. In approximate order of appearance, they include Alan Blinder, Bryan Boulier, Larry Summers, Julio Rotemberg, Matthew Shapiro, David Runkle, Avery Katz, Bob Barsky, Steve Zeldes, Jeff Miron, Mike Whinston, John Campbell, Andy Abel, Richard Zeckhauser, David Romer, Larry Ball, Miles Kimball, David Weil, Olivier Blanchard, Su-santo Basu, Robert Barro, Xavier Sala-i-Martin, Bob Hall, Niko Canner, and Doug Elmendorf. Some of these co-authors were my mentors, others were my contemporaries (often fellow students at MIT), and still others were students of mine at Harvard. In recent years, I have done most of my research with these co-authors.

Why are co-authors so important for the way I work? One reason is found in Adam Smith's famous story of the pin factory. Smith observed that the pin factory was so productive because it allowed workers to specialize. Research is no different--it is just another form of production, Doing research takes various skills: identifying questions, developing models, proving theorems, finding data, analyzing data, expositing results. Because few economists excel at all these tasks, collaborating authors can together do things that each author could not do as easily on his own. In manufacturing knowledge, as in manufacturing pins, specialization raises productivity. (The puzzle is why Adam Smith chose to ignore his own analysis and write The Wealth of Nations without the benefit of a co-author.)

The second reason I work with co-authors is that it makes my job less solitary. Research and writing can be a lonely activity. It is easy to spend endless hours with pad and pencil or in front of a computer without human contact. Some people may like that kind of work, but not me. Arguing with my co-authors makes my day more fun.

The third reason I work with co-authors is the most important: a good co-author improves you forever. In the most successful collaborations, both co-authors learn from the experience. A co-author can help you expand your knowledge, improve your skills, and expose your biases. Even after the collaboration is over, you take these benefits with you to future projects. To a large extent, as I have grown older, my co-authors have become my mentors.

Rule No. 3: Have Broad Interests Throughout my life, I have been blessed with broad interests. (Or, perhaps, I have been cursed with a short attention span.)

As a child, I had numerous hobbies. I collected coins, stamps, shells, rocks, marbles, baseball cards, and campaign buttons. For pets, I had turtles, snakes, mice, fish, salamanders, chameleons, ducks, and, finally, a cocker spaniel. In high school, I spent my time playing chess, fencing, and sailing. I have long since given up all these activities (although I do have a border terrier named Keynes).

As a college student, I committed myself to a new major several times each semester, alternating most often among physics, philosophy, statistics, mathematics, and economics. After college my path was indirect and largely unplanned. In chronological order, I spent a summer working at the Congressional Budget Office, a year studying at the MIT economics department, a year studying at Harvard Law School, a summer working at a law firm, a year working at the Council of Economic Advisers, a second year at MIT finishing my PhD, another semester studying at Harvard Law School, and then another semester at MIT, this time as an instructor teaching statistics and microeconomics. In 1985, I gave up my studies in law and became an assistant professor at the Harvard economics department, where in my first year I taught principles of economics and graduate macroeconomics.

Remarkably, I have been at Harvard now for about a decade. Harvard is a wonderful place to work. Yet I often get the itch to leave, just for the sake of doing something different. One thing that keeps me at Harvard is the proximity of the National Bureau of Economic Research. Every year the NBER holds dozens of conferences on various topics with prominent economists from around the word. Having an office at the NBER is a bit like moving to a new university every few days.

My broad interests (short attention span) help to explain my diverse (incoherent) body of work. My research spans across much of economics. Within macroeconomics, I have published papers on price adjustment, consumer behavior, asset pricing, fiscal policy, monetary policy, and economic growth. I have even ventured outside of macroeconmics and published papers on fertility with imperfect birth control, the taxation of fringe benefits, entry into imperfectly competitive markets, and the demographic determinants of housing demand. None of this is part of a grand plan. At any moment, I work on whatever then interests me most.

Coming up with ideas is the hardest and least controllable part of the research process. It is somewhat easier if you have broad interests. Most obviously, broad interests give you more opportunities for success. A miner is more likely to strike gold if he looks over a large field than over the same small field over and over again. More important, thinking about one topic can generate ideas about other topics. I started thinking about menu costs and macroeconomic price adjustment, for instance, as I sat in a law school seminar that was discussing monopoly pricing and antitrust policy. Research ideas pop up in unexpected places.

Of course, breadth has its costs. One is that it makes writing grant proposals more difficult. I am always tempted to write, "I want to spend the next few years doing whatever I feel like doing. Please send me money so I can do so." Yet, in most cases, those giving out grant money want at least the pretense of a long-term research plan.

The greatest cost of breadth, however, is lack of depth. I sometimes fear that because I work in so many different areas, each line of work is more superficial than it otherwise would be. Careful choice of co-authors can solve this problem to some extent, but not completely. I am always certain that whatever topic I am working on at that moment, someone else has spent many more hours thinking about it than I have. There is something to be said for devoting a lifetime to mastering a single subject.

But it won't be my lifetime. I just don't have the temperament for it.

Rule No. 4: Allocate Time with Care This is a rule of thumb I have been slow to learn. I used to go to every school that invited me to give a seminar, comment on every paper that a conference organizer asked me to discuss, referee every paper that a journal editor sent me, write every letter of recommendation that a department chairman requested of me, and sit on every committee that a dean asked me to attend.

But no more. Over time, the number of such requests has increased exponentially. Within a few years of going on the Harvard payroll, the cost of saying yes became intolerable. I came to realize that too much professional responsibility can be irresponsible, for it takes time away from the most important tasks--teaching and research. I now turn down the overwhelming majority of offers from seminar organizers, conference organizers, journal editors, department chairmen, and deans.

Deciding which research projects to pursue is the most difficult problem I face in allocating my time. I find it almost impossible to predict how any project will turn out before it is done. And even when I have finished one of my papers, I cannot predict with much accuracy how other people (such as editors and referees) will react to it. My strategy, therefore, is to choose research topics based- on what interests me most and, to some extent, on whether I have a good co-author who shares my enthusiasm. Sometimes I work on a topic for a while and decide that I have nothing new to say. I then force myself to remember the irrelevance of sunk costs and move on to another topic.

One way that I spend quite a bit of time is writing textbooks. I have written an intermediate-level textbook on macroeconomics, which is now in its second edition, and I am now in the process of writing a textbook on the principles of economics. Writing a textbook is a lot of work, and I am sometimes asked why I choose to spend my time this way. So let me explain.

Textbook writing is a form of teaching. As such, it has all the pluses and minuses of teaching. The major minus is that it takes time. And time is an academic's most valuable resource.

Despite the cost, I view textbook writing, like classroom teaching, as a good use of my time. One benefit is pecuniary. Few people in the world earn a living just creating knowledge. Most academics spend some of their time imparting knowledge as well. Giving lectures is one way of imparting knowledge; writing textbooks is another. So far, I have been able to make enough money imparting knowledge to students that I have not had to spend time on other activities, such as paid consulting, to put food on the table.

Of course, the most immediate benefit of classroom teaching and textbook writing is that they allow you to mold the minds of students. Economics is not a straightforward discipline like Newtonian mechanics or Euclidean geometry. Whenever you teach economics, you have wide latitude in choosing what material to include and how to present it. In making these choices, you give your own "spin" to the subject and help determine the views of your students. Although classroom teachers and textbook writers share this responsibility, textbook writers reach a larger audience. For those who want to bequeath their view of economics to the next generation, textbooks are the most efficient medium. Indeed, because textbooks are so important in shaping the field, many of the most prolific writers in academic journals are also textbook authors: Samuelson, Baumol, Blinder, Stiglitz, Barro, Dombusch, Fischer, and on and on,

A less obvious benefit of classroom teaching and textbook writing is that they stimulate ideas for research. Whenever you have to explain something to someone, either in person or on a printed page, you have to think it through more thoroughly than you otherwise would. Preparing a lecture or drafting a textbook chapter reveals holes in your understanding. And, sometimes, as you try to fill these holes, you get ideas for research. Put simply, imparting knowledge and creating knowledge are complementary activities. That is why these two forms of production take place in the same firms, called universities.

The final benefit to spending time writing textbooks is that it makes you a better writer. But that brings me to my next topic.

Rule No. 5: Write Well I think of myself as a mediocre writer. I do not come by my mediocrity naturally. It is the result of hard work and determination. This may seem like a small accomplishment, but I reassure myself with the fact that most economists do not live up to this standard.

Economists tend to underestimate the value of good writing. The reason, I believe, is that we like to think of ourselves as scientists. Scientific truths are as valid in run-on sentences as in well-written prose, so why bother trying to write well? Of course, no one would actually endorse bad writing, but this subconscious attitude pervades the profession and explains why economics is a more dismal science than it needs to be.

Despite our profession's bad attitude toward writing, good writing is in fact extraordinarily helpful to achieving success. Everyone knows that Robert Solow and Robert Lucas are important economists. But they are also superb writers, and this fact helps explain their prominence.

Whenever a person sits down to write something about economics, he is engaged in a form of joint production. Each article has two key attributes: style and substance. For producers of articles, style and substance are substitutes. The more time is spent avoiding the passive voice and replacing a "which" with a "that," the less time is left to spend thinking new thoughts about the economy. But if you want to succeed as a producer, you have to think about your consumers. For consumers of articles, style and substance are complements. When I see an article by Solow or Lucas, I want to read it, not just because I will learn something about economics, but also because I will have fun doing so. An article that offers both style and substance is far more appealing than an article that offers one without the other. So if you want to sell your substance, you have to worry about your style. In other words, if you want to be read widely, you have to write well.

Writing is a craft, like carpentry. Some people are naturally better at it than others. But anyone can get better at it by devoting enough time and effort.

The first step to writing better is deciding to write better. After that, it is like acquiring any skill. Just as you can learn how to run regressions by reading a RATS manual, you can learn how to write better by reading books on style. I often recommend Strunk and White's The Elements of Style to my students, and I am surprised at how many have never heard of it. (It is the perfect book to leave in the bathroom. Whenever you have a spare minute, open it to a random page and start reading.) I also recommend that students read William Zinsser;s On Writing Well to learn how to write and Donald McCloskey's The Rhetoric of Economics to learn how to persuade.

Becoming a good writer also takes practice. Reading the RATS manual will tell you how to run a regression, but you cannot easily run a regression after just reading about it. You have to turn on the computer and try it several times. You see what mistakes you make, what bugs show up unexpectedly, what things the manual forgot to tell you. The same is true with writing. The more you write, the better you get. When I look back on my own education, one thing that stands out is how often I had to write in the (private) high school I attended. I always had some writing assignment hanging over my head. At the time the school's policy seemed oppressive, but now I am grateful for the oppression. It prepared me perfectly for my current job.

Writing well is hard work. It requires that you revise, revise, and revise. Then, when you think you are done, you should revise again. Good writing is fun to read, but it is often not fun to do. (I once asked John Kenneth Galbraith the secret to his success as a popular writer. He said that he revises everything many times. Around the fifth draft, he manages to work in the touch of spontaneity that everyone likes.)

Fortunately, modem technology has made writing much easier. I write directly in Wordperfect. Pen, paper, and secretary are not necessary, which surely makes me more productive. But modem technology has also made it easier for people to produce bad writing. The supply of good writing and the supply of bad writing have both increased over time. The demand for bad writing remains low, however, so in equilibrium there is not much reward for producing it.

By contrast, good writing has substantial rewards. Writing something well attracts readers and gives your ideas a better chance to be heard. But there is also another payoff: good writing brings personal satisfaction. An author should get pleasure from looking back and finding that he has presented his ideas well. I do not like writing, but I do like having written.

Rule No. 6: Have Fun A book I read long ago revealed to me the secret to a happy life: find out what you like to do, and then find someone who will pay you to do it.

I learned this secret as a teenager. At the time, I liked racing small sailboats. So, when I looked for my first summer job, I found one giving sailing lessons. (My employer charged $15 for a one-hour lesson and paid me the minimum wage of $2.25. This was my first lesson in the economics of monopolistic competition.) Yet I knew that this advice would not always be easy to follow. I had no idea how to find someone to pay- me to race sailboats for the rest of my life, and this was a source of some adolescent distress. Luckily, my tastes changed as I aged.

I now keep the secret to a happy life in mind when selecting topics for research. Editors and conference organizers often invite me to write papers on specific topics of their choosing. I turn down most of these offers. (This essay is one of the few exceptions.) Unless the editor happens to propose a topic in which I am already interested, I will not enjoy writing the paper and, most likely, will not do a good job. My approach to research is to decide first what I want to think about. I then see if I can get someone to publish the result. If my current interests happen to coincide with a conference someone is organizing, that's great, for the conference is a convenient outlet. And a conference invitation might help me to choose among-several projects that I have in the back of my mind. But the most important question for me when beginning any project is whether the topic gets me excited.

Graduate students starting work on their dissertations often ask me for strategic advice. What are the hot research areas? What topics will get them jobs at the top universities? It is easy to understand why students ask these questions, but these are the wrong questions for someone embarking on a research career. I tell students that they should be asking themselves more personal questions. What would they like to learn about? What do they observe in the world and find puzzling? What topics get them excited?

Doing research is not like digging a ditch. A person can dig a perfectly fine ditch without enjoying his job for a Minute. By contrast, research requires a certain passion about the topic being studied. Passion goes hand in hand with creativity. No one can manufacture this passion for strategic reasons of career advancement.

Most people who pursue an academic career do so because they are fascinated by their subject. It is for this reason that professors report among the highest rates of job satisfaction of all professions. Professors have found what they like to do, and they have found someone to pay them to do it.

~~~~~~~~

By N. Gregory Mankiw

Read more!

Research Experiance of Dixit (from mit.bbs)

MY SYSTEM OF WORK (NOT!)

Contents My Own Experience of Research On Choice of Topics On Habits of Work On Writing A Concluding Word Notes

Among the signals of approaching senility, few can be clearer than being asked to write an article on one's methods of work. The profession's implied judgment is that one's time is better spent giving helpful tips to younger researchers than doing new work oneself. But of all the lessons I have learnt during a quarter century of research, the one I have found most valuable is always to work as if one were still twenty-three. From such a young perspective, I find it difficult to give advice to anyone. The reason why I agreed to write this piece will appear later. I hope readers will take it for what it is--scattered and brash remarks of someone who pretends to have a perpetually juvenile mind, and not the distilled wisdom of a middle-aged has-been.

Writing such a piece poses a basic problem at any age. There are no sure-fire rules for doing good research, and no routes that clearly lead to failure. Ask any six economists and you will get six dozen recipes for success. Each of the six will flatly contradict one or more of the others. And all of them may be right--for some readers and at some times. So you should take all such suggestions with skepticism. Give a good try to any that appeal to you, but don't fear to disregard all the rest.

There is also the problem of judging the target audience. What works for academic research is not best suited for policy or consulting research, and the right strategy for advancing the frontier of research is not the same as that for later work of consolidation or synthesis. I will assume that the readers of these essays are actual or potential academic economists with high ambition; they aim to excel in whatever area of research they choose, and are looking for good habits to speed their journey. In short, I am assuming that the readers hope for success at the top levels of the research community.

These general difficulties are compounded by my own limitations. First, I am a theorist, albeit of a relatively applied kind. That is to say, I build mathematical models to address specific issues and contexts of economic interest, rather than abstract systems of general and overarching significance. And I try to get specific results from the models (What cause has what effect?) rather than prove theorems (Does equilibrium exist, and is it unique?). What works for me is governed by what I am trying to accomplish; the same approaches and techniques may not suit the more abstract theorists or the empirical economists.

My second limitation is even more severe. I have always worked on the next problem that grabbed my interest, and tackled it using whatever approaches and techniques seemed suitable, never giving a thought to how it might fit into an overall world-view or methodology. It is hard for me to evaluate such an unsystematic and unphilosophical approach, and even harder to give any advice based on it. But I shall try.

My Own Experience of Research Readers of these essays are surely not too interested in the drab and dreary lives of economists for their own sake; they are in search of research methods they can emulate. But one's advice is colored by one's experiences, and I owe the reader a brief statement of the reasons for my biases.

Most of us spend hours discussing at which restaurant to have dinner, and make decisions like what career to pursue and whom to marry instinctively in an instant. So it was with my entry into economics. I got my first degree in mathematics, and had just started a Master's in Operations Research, when I was converted to economics by a chance conversation with Frank Fisher. He should get all the credit, or the blame.

I started on my research career in 1968, at a time of turmoil in the academic world of Europe and the US. The prevailing atmosphere was decidedly left-wing and anti-establishment, and research was almost required to be "relevant." Most theorists were affected by this atmosphere, and I was no exception. Important topics included problems of less developed countries, urban problems, and environmental problems.[1] I dabbled in all of these.

Looking back on those years, much of the "relevant" research in economics left little lasting mark on the subject. Problems of less developed countries and urban areas proved so decidedly political that good economic advice would have achieved nothing even if we had been able to give it. No, the topics that proved to have lasting value in economics were quite different, for example the theory of rational expectations, the role of information and incentives, and later in this period, game theory. In the early 1970s much of this work seemed abstract and irrelevant, and would have been called "politically incorrect" had that phrase existed in those days.

My own work also met the same fate. My "relevant" work is mostly and justly forgotten.[2] What has come to be regarded as a success--for example the theory of product diversity in monopolistic competition, the theory of entry-deterrence in oligopoly, a reformulation of the theory of international trade, and some recent work on irreversible investment--was not motivated by any sense of relevance, or any high-minded desire to do good. It is almost embarrassing to think back on how I came to work on some of these topics.

The book on international trade grew out of a lunchtime conversation with Victor Norman. He knew a fair bit about the subject, I knew almost nothing, but both of us knew a lot of duality theory and had a sense it might be useful in simplifying some of trade theory. We decided to learn by doing, and spent so long at it that we had to write a book. As we went along, more than half of the time we found that someone else had been there before. But it was much more fun to do it ourselves.

The model of entry deterrence in oligopoly came from an uneasy feeling that the accepted theories--Bain-Sylos, and even Spence--were not doing it right. At the time subgame perfectness had just made its appearance in the game theory literature, but I was in rural England, far removed from the centers of game theory like Stanford, and had never heard of the concept. So I had to work it out from scratch, and that took a surprisingly long time. The breakthrough came when I had by mistake gone to the airport far too early, and had to kill a couple of hours. Once the right idea came, everything worked out really fast. Since then I have often deliberately got to airports too early, but alas, with no success.

Much of this work was received favorably by some existing specialists in the fields, but got puzzled and negative reactions from others: "Optimum product diversity? Surely the market finds the optimum. Monopolistic competition? That's a dead end." "Duality? What's wrong with the way we have always done things?" For years Ron Jones dismissively referred to the group working on oligopoly in international trade as "imperfect competitors." By now I expect a "long and variable lag" between the time I work on something and the time enough others find interest or use in it. But I have learnt the importance of trying to shorten the lag by conveying my ideas simply and clearly.

In this I am but one minor member of an extremely distinguished group. For example, William Sharpe straggled to get his now-famous CAPM paper published, and recalls the reaction even after its appearance in print: "I knew . . . [t]he phone would start ringing any moment. After one year, total silence. Nobody cared. It took quite a while."[3]

As you can see, my approach to research is too opportunistic to have a constant direction. But taking stock of it for the purpose of writing this piece, I could see a recurrent if not dominant theme. Scale economies and sunk costs keep appearing in my papers with great regularity. Imperfect competition is the norm, and market equilibria are not socially optimal (but government interventions have more subtle effects than naive intuition would suggest, and may actually make matters worse). And therein lies an irony. The left-wing critics of the late 1960s and 1970s, who influenced many youngsters when I started out, reserved their strongest criticism for the perfectly competitive equilibrium of the neoclassical system. Of course they did little by the way of offering a viable alternative. It has been the unexciting incremental work, to which I have contributed a little, that has built into a major shift in our understanding of how the economic system operates when the assumptions of neoclassical economics fail.

That is enough autobiography, and more than enough self-justification. For the rest of the article, I shall elaborate and paraphrase my experience into statements of what I have found to be good work habits. I will find it convenient to express these as items of advice, but let me repeat my earlier caution to the readers--be skeptical, pick what you think might suit you, and discard the rest.

On Choice of Topics * My most important advice here is stark and politically very incorrect: Don't give too much weight to the social importance of the issue; instead, do what captures your intellectual interest and creative imagination. This is not to deny the importance of paying attention to the real world. Nor is it to say that abstract theory is necessarily more valuable than applied work. Nothing could be farther from the truth. But I do believe that mere relevance of an issue will not guarantee good research unless you have a genuine drive to work on it. If not, leave it to someone else. Good work on an apparently unimportant problem will have more long-run value than mediocre work on one of greater intrinsic importance. And one's judgment of importance can always be wrong; concepts of relevance can change over time.

Of course if you find genuine passion for an issue of real social importance, count yourself twice blessed.

* How can you know if you do have the real drive to do research on a particular topic? Perhaps the surest sign is that the work is fun. Richard Feynman, in a wonderful collection of anecdotes from his life ("not an autobiography," he insisted) gives a classic example of this.[4] Some students in the cafeteria were tossing around a dinner plate like a frisbee. It was wobbling, and the red Cornell medallion on the plate seemed to be revolving faster than the wobble. Feynman set out to calculate the relation between the two rates and found a remarkably simple two-to-one ratio. He showed his work to a senior colleague.

'He says, "Feynman, that's pretty interesting, but what's the importance of it? Why are you doing it?"

"There's no importance whatsoever. I'm just doing it for the fun of it." . . . And before I knew it . . . I was "playing"-working, really. . . . It was effortless.

There was no importance to what I was doing, but ultimately there was. The diagrams and the whole business that I got the Nobel Prize for came from that piddling around with the wobbling plate.'

Feynman uses a very revealing word: "playing." If your work is as enjoyable to you as play, that is a good sign that the topic suits you.

Looking over what I have just said, I realize that I am advocating something very radical: not only a non-system, but also a non-system for non-work. But what did you expect from someone of twenty-three?

* Every bright student who passes his/her general examinations sets out to revolutionize the subject. But revolutions are not best made by setting out to make them. In Thomas Kuhn's terminology, scientific revolutions are the consequences of attempts to resolve anomalies that are observed in the course of normal science. And the best way to notice anomalies is to do normal research.

* Discover your best "distance." Some people are good sprinters in research. They can very quickly spot and make a neat point; they do this frequently, and in many different areas and issues. Hal Varian and Barry Nalebuff are two of the best sprinters I know. In the same metaphor, others are middle-distance runners. In fact most economists are at some point in this broad category. A few, for example Robert Lucas and James Mirrlees, are marathoners; they run only a small number of races, but those are epics, and they get the most (and fully deserved) awe and respect. In contrast, the profession seems to undervalue sprinters. But each kind of work has its own value, and the different types are complements in the overall scheme of things. Progress of the subject as a whole is a relay race, where different stretches are of different lengths and are optimally run by different people. Find out where your comparative advantage lies.

* Many ideas, and techniques for theorizing, will come to you by accident. But don't wait for such accidents to happen; encourage them. Always be on the lookout for examples, questions etc that relate to what you are doing, or something you worked on once but set aside. A newspaper article or a current affairs program or a chance remark by a colleague can get you started. A totally unrelated theoretical article may use a technique that proves useful for your problem, and gets you re-started on something that had stalled. Seemingly far-fetched analogies turn out to have some deep basis. Therefore you should keep all of your work in your semi-active memory all of the time--the work in progress as well as that not making progress.

* Learn to manage your time. When asked to contribute to a collective volume, or present a paper at a conference, unless the assigned topic happens to coincide exactly with your interests, follow the Nancy Reagan strategy: "Just say no." You will invariably find the demands of such assignments crowding out the time that you could have spent on ideas of much greater intellectual interest to you. (In fact I took on the task of writing this article just to get that out.) Stick to what you would best like to do; if you are successful, some years later people will be holding conferences on your topic. (Of course by then you will be interested in something else.) In the meantime, you will have much more fun working on something that you really like. And even the material rewards of a successful frontier research article easily exceed the honoraria of ten conference articles of topical interest.

There are people who can turn a conference assignment into real research. Or to be accurate, there is one such person--Paul Krugman. Unless you have that very rare skill, get your priorities straight.

On Habits of Work * Management of your time is again of paramount importance. This is especially true when on occasion you are forced (or just irresistibly tempted) to violate the Nancy Reagan Strategy and take on a conference-type assignment. Then I recommend the Nike strategy: "Just do it." Don't procrastinate to the deadline. If you do, you will waste a great deal of time all the while, thinking about the assignment and its impending deadline. You will also expend a lot of mental energy feeling weighed down by the task. Much better to get it out of the way as quickly and effortlessly as possible, and get back to the real stuff.[5]

* On the other hand, when doing frontier research of real intellectual importance and challenge, do not be afraid to spend a lot of time thinking vaguely, or even "day-dreaming" around the subject. This time is not wasted. All the associations you ponder, and all the calculations you try for a few lines and abandon, will prove a useful input to the process that ultimately leads to the answer.

* Having posed the question and worked on it for a while, give the subconscious a chance. Perhaps the best advice on this comes from the mathematician J. E. Littlewood, in his lovely article, "The Mathematician's Art of Work."[6] He distinguishes four phases in creative work: preparation, incubation, illumination, and verification. "In preparation, [t]he essential problem has to be stripped of accidentals and brought clearly into view; all relevant knowledge surveyed; possible analogues pondered. It should be kept constantly before the mind during intervals of other work. . . Incubation is the work of the subconscious. . . Illumination, which can happen in a fraction of a second. . . . almost always occurs when the mind is in a state of relaxation, and engaged lightly with ordinary matters." Littlewood recommends "the relaxed activity of shaving" as a fruitful time for illumination; I shudder to think how much more David Kreps and Paul Krugman would have accomplished if they had known this.

* In our profession it is customary to stress the importance of economic intuition, and deride abstract or formal thinking. I have found this to be right on balance, but not to the point of dogma. People and problems vary in the kind of thinking that suits them best. For example, it appears that John yon Neumann had a very abstract kind of mind. He once advised a co-worker: "Oh no, no, you are not seeing it. Your kind of visualizing mind is not right for seeing this. Think of it abstractly. What is happening [on a photograph of an explosion] is that the first differential coefficient vanishes identically, and that is why what becomes visible is the trace of the second differential coefficient."[7] How many of us, heating such an explanation from a colleague or a student, would have admonished them to "be more intuitive?"

* Keep a "portfolio" of problems to work on. If you are not making progress on one, switch to another. You will not only diversify your risks, but also increase your chances of success on each, because your mind will stay fresher and you will feel less depressed about the lack of progress on one problem. But don't switch too rapidly; if a problem is at all challenging, less than a month's concentrated thinking about it may not be good enough.

* Joint research is becoming more common in economics, and that is a good thing. A good research collaborator is worth any number of casually interested readers of your papers. The close but sympathetic criticism at an early stage that comes from a fellow worker helps you avoid many blind alleys, or wrong tacks from which you might otherwise never recover. As Francis Crick put it, "The advantage of intellectual collaboration is that it helps jolt one out of false assumptions."[8] You and your ideal co-author will have enough overlap to give both a common frame of reference and language for thinking, but enough difference to generate real synergy and complementarity rather than mere duplication.

* Reserve your best and most alert period of the day for real research, and use your tired, dull or slack stretches for correspondence, meetings, administrative chores etc. Alas, this is often not possible. Keep in mind, too, the possibility that your best period changes with the seasons, age, etc. I have heard Paul Samuelson claim that for most people a switch occurs at around 35 years of age: morning becomes a better time for research instead of late at night. My own experience confirms this.

* Continue revising your papers to improve them, but not forever. The Austrian capital theory that you learnt as a dry textbook model has practical application. Papers should he improved only to the point where the rate of improvement equals the rate of interest. The latter rate will vary over your life-cycle, but striving for absolute perfection is wrong for most people at most times. From a private perspective, it will delay the spread and impact of your work too much, and risk pre-emption. From a social perspective, public release of something that is less than perfect has value; it may be someone else's comparative advantage to contribute the next step of improvement.

* Read other people's papers either seriously,or not at all. When you read them seriously, read them as you read papers when you were a graduate student, checking all the details and questioning everything. This is a good way to get new research ideas of your own. I owe my own understanding of the importance of this principle to Richard Feynman. He describes how he came to discover the law of beta decay.[9]

'At that particular time I was not really quite up to things. Everybody seemed to be smart, and I didn't feel I was keeping up. . . . At one point there was a meeting in Rochester, . . . and Lee was giving his paper on the violation of parity. . . . I was staying with my sister in Syracuse. I brought the paper home and said to her, "I can't understand these things Lee and Yang are saying. It's all so complicated." "No," she said, "what you mean is not that you can't understand it, but that you didn't invent it. You didn't figure it out your own way, from hearing the clue. What you should do is imagine you're a student again, and take this paper upstairs, read every line of it, and check the equations. Then you'll understand it very easily." '

She was right. Not only did Feynman understand the paper, but he remembered something he had done a while ago, used that method to simplify Lee's solution, and forged ahead to develop the whole new theory.

Oddly enough, when I read this I was in a somewhat similar state of mind with regard to the literature on trade policy with asymmetric information, and the same recipe worked for me.[10]

On Writing * My first suggestion is: Keep it simple. The temptation to show one's technical wizardry is overwhelming, particularly for the fresh Ph.D. Resist it. It will only make your paper less easy to read, and reduce its impact. If an idea can be conveyed in a simpler way, without spelling out every epsilon and delta, do so. Littlewood says of Jordan that if he wrote an article with only four symbols they would be called a, M'3, epsilon2, and II"1, 2 instead of a, b, c, d; don't be like that.[11] If needed for completeness, put the more formal proof in an appendix. However, I find totally unacceptable the current and growing practice of many papers in economic theory, which merely state the results in the text without any explanation at all, and then relegate the proofs to an appendix.

I said earlier that pure economic intuition may or may not be the right way to think in research. Its importance increases when one writes research results, and even more when one talks about them, particularly if the intended audience is larger than that of specialists in a very narrow area. (Many fresh Ph.D.'s giving job talks do not realize the importance of a simple and intuitive exposition, and this costs them dearly.)

* My second suggestion is: Keep it short. In this I agree with Piet Hein, the Danish scientist turned poet who wrote aphoristic verses called Grooks. He preferred writers

'who find their writing such a chore they only write what matters.'

But this seems a lost cause. Over the last two decades the average length of economics papers has increased quite a lot. Advances in word-processing technology have greatly reduced the cost of producing words, but not the cost of producing ideas, with the result economists should expect--massive substitution.

My ideal is neatly captured in a question Frank Hahn posed to an author. As an editor of the Review of Economic Studies, Hahn asked the author to cut down his paper from 40 pages to its essential core of three pages. When the author wrote a long and indignant letter, Hahn responded in two sentences: "Crick and Watson described the structure of DNA in three pages. Kindly explain why your idea deserves more space." An ideal that, alas, neither I nor Frank Hahn nor anyone else seems to come close to.

* Listen to referees: Referees may be prejudiced, they may be hurried, but they are almost never stupid. If you are doing innovative work, be prepared to meet bias, and be prepared to meet careless dismissal. Give such reports due consideration--even they may contain useful tips for revision--but if you have basic confidence in what you are doing, press on. If you meet sheer incomprehension, however, take that as a sign that your writing has failed. Clarify, if necessary overhaul the whole notation of your formal model, and try new drafts on colleagues and students, until you communicate better. I come across many economists who constantly complain that "referees don't under-stand them." My inner response is the same as Tom Lehrer's: "If a person can't communicate, the very least he can do is to shut up."

* There are conflicting considerations on how hard to sell your work. On the one hand, if you don't sell your own work, the chances are that no one else will. Littlewood has the mot juste once again:[12] "He that bloweth not his own trumpet, his trumpet shall not be blown." On the other hand, excessive claims about the importance of your work will get you a bad reputation in the profession, and will jeopardize the reception of your future work. I prefer to claim a little less for my work than I feel it deserves.

If you must exaggerate, do so in a skillful way. Joseph Schumpeter claimed that he set out to become the best horseman in Vienna, the best lover in Europe, and the best economist in the world, and had achieved two out of the three. This is brilliant exaggeration--anyone who actually knew Schumpeter's prowess in one of the three things would give him the benefit of the doubt and assume that he had excelled in the other two.

A Concluding Word I have saved for the end the most important lesson I have learned from my experience, and which I believe has very general validity. Maintain a youthful sense of freedom to choose problems and the directions of work on them. Imagine yourself at twenty-three, not yet labelled or confined to a particular "field," and not yet pressured to produce something quickly for the approaching tenure review. Try to preserve this mental frame in your research, even as your body, and the part of your mind dealing with other matters, continue to age and decay.

Unfortunately, in the US most academics do not regain this freedom until they are thirty-five, by which time it is too late for many of them to be twenty-three. Their research brain is beyond rejuvenation, and it is time for them to leave the research frontier and join the conference circuit or the policy community. My reaction as a theorist echoes what Clemenceau said on heating that the famous pianist Paderewski had become the President of the newly founded Polish Republic: "What a come-down!"

Notes 1. And for reasons that escape me, quite abstruse arguments in capital theory that acquired inexplicable ideological significance. But that fashion died, as it richly deserved to.

2. I wrote one paper in urban economics--a model of the optimum size of a city trading off scale economies in production and congestion diseconomies in transport--that achieved some success. I like to think that even now, when theoretical urban economists meet for a beer at a conference, someone might remark: "Wonder what became of that guy Dixit. He wrote one paper that wasn't bad, and was never heard from again. I guess some people just don't have staying power."

3. Quoted in Peter Bernstein, Capital Ideas, The Free Press, 1992, p. 199.

4. Richard Feynman, "Surely You're Joking, Mr. Feynman!", New York: Norton, 1985, pp. 157-8.

5. I have to confess that I have not optimized my own time as I advise you to, and that I have too often violated both the Nancy Reagan Strategy and the Nike Strategy. These are merely what in the light of hindsight I wish I had done consistently.

6. Rockefeller University Review, 1967, reprinted in Littlewood's Miscellany, ed. Bela Bollobas, Cambridge University Press, 1986.

7. Norman Macrae, John yon Neumann, New York: Pantheon Books, p. 211.

8. Francis Crick, What Mad Pursuit: A Personal View of Scientific Discovery, London: Penguin Books, 1990, p. 70.

9. Feynman, op. cit., pp. 227-8.

10. But, as with the Nancy Reagan and Nike strategies above, I must confess that I have not followed my own advice on serious reading as consistently as I should have.

11. P. 60 of the Bollobas (ed) book cited above. Incidentally, on pp. 49-53 of the same book, Littlewood gives a beautiful example of how not to, and how to, write up a mathematical argument; I urge every young theorist to read it and absorb its lesson.

12. In ed. Bollobas, op. cit., p. 158.

~~~~~~~~

By Avinash Dixit[*]

Professor of Economics, Princeton University.


Read more!

关于国内公民维权案件的一篇文章,转自mit

县委书记该有什么名誉权

时间:08-11 09:02 作者: 何兵 新闻来源:检察日报

我的家乡安徽,新近发生了一起万人瞩目的诉讼案。原告张西德系安徽临泉县委前任书记。被告陈桂棣、春桃是安徽学界名人,他俩花费数年心血完成的《中国农民调查》一书第四章讲述了临泉县白庙镇王营村村民因农民负担过重多次集体上访,与地方政府发生激烈冲突事件的来龙去脉。作者认为,时任临泉县县委书记的张西德,“负有不可推卸的责任,扮演了极不光彩的角色”。

张西德同志当然不这么看。他义愤填膺地说,相关内容“不仅严重失实、胡编捏造,而且指名道姓地对原告的人格、形象进行丑化,对原告的名誉进行百般损害”。张西德同志决心拿起法律武器,为自己,为临泉县委、县政府及相关机关讨回公道。

张西德同志能够为县委、县政府和他本人讨回公道吗?这是一个非常有意义的案件。我们可以将这个案件的法律问题分解成两部分。其一是,县委、县政府以及其他国家机关有无名誉权?其二,张西德同志能够因为别人批评他的执政能力、措施以及个人品行,而主张个人名誉受损吗?我认为,不能。

首先,国家机关没有名誉权。民法上的名誉权是保护公民、法人在民事活动中的民事权益,而县委、政府的公共管理活动不是民事活动,是党务和政务活动,因此,不受民法上的名誉权保护。其背后的道理在于,在法制社会里,批评政府是人民的基本权利和神圣义务。只有让人民毫无顾忌地批评政府,才可能将政府牢牢地置于人民的监督之下,使人民的政府真正成为为人民的政府。

由于人民对政府的信息掌握不可能完整无缺,由于人民对政府的指责不可能完全客观公正,因此,法律必须容忍人民对政府错误的、不公正的批评。如果法律对人民进行苛求,如果一旦人民的指控出现事实错误和判断错误,就要被追究民事责任,人民将噤若寒蝉;人民纵使对政府不满,也只能张口结舌,望政府而兴叹。正是基于对人民批评政府的权力充分保护,我国宪法规定:“中华人民共和国公民对于任何国家机关和国家工作人员,有提出批评和建议的权利;对于任何国家机关和国家工作人员的违法失职行为,有向有关国家机关提出申诉、控告或者检举的权利,但是不得捏造或者歪曲事实进行诬告陷害。”如果本案中,张西德同志胜诉,这意味着什么呢?这意味着法院在修改宪法,将宪法规定的“有提出批评和建议的权利”修改为“有提出正确的批评和建议的权利”。法院有这个权利吗?没有!

深入研究之后,我们还会发现,即使县委和县政府有名誉权,作为前任县委书记的张西德同志也没有这个诉权,这个诉权归属于县委、县政府,而不归属于张西德同志个人,毕竟个人不代表党和政府。

本案原告张西德强调:“我虽然是一名政府官员,但我有权维护自己的名誉权不受侵害。”这涉及到本案的第二个问题,即人民对官员个人的执政行为提出批评,官员可以主张个人名誉损失吗?答案是,很难。在给出我的理由之前,请看看美国法官是如何分析这个道理的。

案件是这样的。《纽约时报》刊登付费广告,呼吁各界支持马丁.路德.金和南方民权运动,其中有抨击警察局不当行为的言论,部分言论与事实有所出入。当地警察局长沙利文以名誉受损为由提起诉讼。州法院判决原告胜诉,被告赔偿50万元。联邦最高法院在上诉审中一致同意推翻原判,认定公共官员如果不能证明其职务行为的批评者之批评出诸实际恶意,则不能获得损害赔偿。所谓实际恶意即明知为非或不顾事实真实与否之轻率心理状态。

美国布伦南大法官主笔的判决书宣示,宪法第一修正案的核心宗旨,即是使公共官员执行公共权力的行为,接受人民最广泛的批评,而批评政府是公民的一项崇高义务。判决书表示,对于公共问题作无约束力、强而有力、公开的讨论是国家对人民所承诺的一项基本原则。另有三名大法官发有协同意见,主张对公众言论予以绝对的宪法保障。法官布莱克认为:“第一和第四修正案不只是限制各州许可政府官员起诉批评赔偿的权力,而是根本上禁止各州行使这种权力。”法官戈登堡说:“依我之见,宪法第一和第十四修正案赋予公民和报业一种批评官员职务行为的绝对、无条件的特权,而不管这种特权的滥用和过分行使可能带来什么危害。这并不是说宪法保护针对政府官员和普通公民的私人行为的侵害名誉言论。纯粹针对私人行为的侵害名誉言论与一个自治社会的政治目的没有关系。”

这一判决阐明这样一个基本的道理,即人民批评政府官员,即使出现错误,即使官员的名誉事实上受到损害,法律也不保护官员的名誉权??除非有人故意陷害。法律对官员为何如此不公?法律在捍卫什么?法律捍卫的是人民对公共事务不受限制的批评权利,而这一权利能否得到保护,直接关系到国家的安危,法律在“丢卒保车”:为了保护公民的言论自由这个“车”,法律必须丢弃了官员名誉权这个“卒”。



Read more!

4/12/2005

How to publish in top journals? (zz)

http://www.econ.iastate.edu/classes/econ555/choi/
发信人: srrbyes (out+of+date), 信区: Economics
标 题: How to publish at top journals, by Choi
发信站: Unknown Space - 未名空间 (Wed Feb 23 12:09:53 2005) WWW-POST

How to publish in top journals?

Kwan Choi

In response to popular demand, this brief note is provided for the benefit
of all academic authors. The original intent was to produce a book of advice,
but time is a scarce commodity and you may have to wait indefinitely for a
book-length summary.
This brief manual provides some useful suggestions for today’s
authors. The goal is to "foster the greatest good to the greatest number of
people."1 If this note is useful to you, please tell your friends about it. If
you follow most of these rules, the probability of obtaining tenure or
promotion may increase significantly. If most authors acquired the basic
skills mentioned here, they would then be competing in terms of the truth,
goodness, and beauty of their ideas, not in terms of cosmetic skills.
Disclaimer
Please note that the advice contained here may not necessarily
improve the chances that your research papers will be published. By
downloading or acquiring a copy of this guide, you agree that:
In no event shall the author be liable for any indirect, incidental,
collateral, exemplary, consequential, or special damages or losses arising out
of your use of rules suggested in this guide.
Note
1. The Urantia Book (p. 1488)
________________________________________
Introduction
Publishing technology has changed drastically in recent years. The
advent of the personal computers and laser printers has lowered the technical
barriers of publication. Economists are now producing more papers than they
were a couple of decades ago. Consequently, top journals are being inundated
with manuscripts.
Journal editors have become extremely risk averse; they are more
concerned with the risk of accepting low-quality articles than with the
possibility of rejecting good articles.

Purpose of this Manual
Ideally, the decision to publish should be based solely on the ideas
contained in the papers. In practice, the decision is affected by other
nonsubstantive and cosmetic factors. If all authors were equally skillful in
presenting their ideas, they would be competing essentially in terms of the
merit of ideas, rather than the art of presentation.
This manual will advise authors on how to prepare papers to improve
their chances for acceptance in top journals.
Why is the journal acceptance rate so low?
• Among the papers submitted to ranking journals, 1/3 or less receive
mildly favorable reports. (This generally depends on the quality of the
journal and the referees.) The rest do not receive favorable recommendations.
• If two referees are employed, the chance that a typical paper of
average quality will get a favorable recommendation from both referees is
about 11% (= 1/9).
• There is no such thing as good luck in publication. Painstaking work,
coupled with careful risk taking, is required for success.
All referees are not equal. Comments of a well-known referee weigh
more heavily than those of a lesser-known referee. You should be aware of
which referee is more important.
When a paper is rejected, the editors paid more attention to the
negative than the positive aspects of your paper.
If you eliminate or reduce the negative elements, the good ideas in
the paper will far overshadow the shortcomings and your paper is more likely
to be accepted.
Why is your acceptance rate lower than others?
• You may lack experience. However, this can be remedied.
• You may need to submit more papers. Volume also increases the
acceptance rate because of learning by doing.
• Identify the cause and act accordingly. There might be biases against
you based on race, sex, nationality, or schooling. For instance, if a
university journal has a reported acceptance rate of 10% but pre-allocates
half the space to its faculty and immediate students, your acceptance rate is
20% if you are in the preferred class, and 5% or lower if you are not.
• You may not be able to eliminate existing biases, but you can avoid
them.
________________________________________
References
The Chicago Manual of Style, The University of Chicago Press, 1982.
Holub, Hans Werner, Gottfried Tappeiner, and Veronika Eberharter,
"The Iron Law of Important Articles," Southern Economic Journal 58 (1991),
317-28.
Horowitz, Ira, "How to Publish Well and Often When You are Unlikely
to Contend for a Nobel Prize," Research Bulletin, Chinese University of Hong
Kong, Issue 3, November 1995.
Hudson, John, “Trends in Multi-Authored Papers in Economics,” Journal of
Economic Perspectives 10 (1996), 153-9.
Laband, David N. and Michael J. Piette, “Favoritism versus Search for Good
Papers: Empirical Evidence Regarding the Behavior of Journal Editors,”
Journal of Political Economy 102 (1994), 194-203.
Liebowitz, S. J. and J. P. Palmer, "Assessing the Relative Impacts
of Economic Journals," Journal of Economic Literature 22 (1984), 77-88.
McCloskey, Donald, The Writing of Economics, Macmillan Publishing
Company, New York, 1987.
Nyaw, M. K. and Eden Yu, "Professor Douglas North's Research
Experience and Advice," Research Bulletin, Chinese University of Hong Kong,
Issue 2, April 1995.
Urantia Foundation, The Urantia Book, 1955, Chicago.
________________________________________
General Publication Strategies
1. Diversify your research portfolio
o Average wait for an acceptance decision = 3 years.
o Average wait for a rejection = 6 to 8 months.
o Survival is more important than glory in the early stages of your career.
o Diversifying the research portfolio is particularly important during the
first five or six years of your teaching career when each publication counts
heavily. Diversify research topics for possible publication.
o If you have a solid hit in one area, then redouble your effort to establish
your name as an expert in that field before you move into another field.
o Writing several papers in a very narrow area is risky. It is like putting
all your eggs in one basket.
o Continuing to write papers in the same narrow area without clear evidence of
success is risky.
2. Concentrate on one or two fields
o Normally, you should not select more than two fields of specialization.
Research economies of scale often may require your undiluted attention in a
single field.
o Choose, at most, two or three focused areas within your field of
specialization. Then pursue those topics until you produce a couple of
publications.
o If you have published no papers in one area for three years, then consider
switching to another topic.
3. Generate one or two papers from your thesis
o You invested two or more years writing your thesis.
o Try to generate a couple of papers from the most important chapters of the
thesis. This is easier than writing a totally new paper from scratch.
o Work jointly with your advisor to help market your papers.
4. Maintain a stock of papers under review constantly
o If the acceptance rate of the top-ranking journals is 15%, one needs about 7
papers under review at all times to have one paper accepted per year at the
targeted journals.
o If your goal is to get 10 papers accepted in the first 5 years of your
career, you need about a dozen papers under review at all times.
o Half a dozen papers should be under review at all times for untenured
authors. This does not mean that you should write 7 new papers each year.
5. Don't put two good ideas in one paper
o Separate them into two papers.
o Do not try to put down everything you know about the subject in one paper.
What will you do next?
o As the paper's length increases beyond 15 pages, the chance of acceptance
shrinks geometrically.
o When a topic is appropriately split into two papers, the probability of
getting at least one of them accepted more than doubles.
o You also will get a paper accepted sooner.
o If x = original length, and p = probability of acceptance, then
p(x/2) = 2p(x) + , where  > 0 and x > 15 pages.
The alpha () factor:
 Editors like short papers.
 The chance that a referee will detect a mathematical error declines.
 Referees will return the report faster.
 The chance that a referee will misunderstand the paper also
decreases.
6. Approach different types of journals
o Sending all papers to top journals is risky.
o Sending all papers to low-quality journals also is unsatisfactory. You will
regret it when the papers are accepted!
o Your curriculum vitae should contain some publications in the top journals.
o Quantity of publications also is important.
o Having three papers in different journals is better than three in one
journal, if the relative quality of the journals is the same.
7. Write clearly
o The main assumptions and results should be explained clearly. If there are
many assumptions, present them together in one place. Do not bury them in long
paragraphs.
o Define every symbol when it is first introduced. Otherwise, the referees
will be frustrated, and you won't get a favorable report.
o If many symbols are introduced to present your model, it is a good idea to
define all symbols together and display them in one place so that the referees
would not waste time hunting for them.
o Clearly state the contributions of the paper, relative to the literature, in
the concluding remarks.
8. Learn word processing skills and master other relevant software
programs
o Be independent of secretaries. They do not work 24 hours a day.
o Word processing skills are particularly helpful when the amount of revision
is minimal.
o Researchers without computer skills will be an endangered species in this
century.
9. Scan current journals
o Keep up with the current literature (e.g., EconLit).
o Using the potential key words, search to see if others have written papers
on the same or similar subjects.
o By not duplicating what others have done, you will save time and effort.
o Subscribe to a couple of journals in your field of interest, rather than
general journals.
o General journals are not cost effective as a source of research information.
Fewer and fewer articles in general journals are relevant for your research.
o Utilize the libraries for other journals.
o Social Science Research Network features news about papers as soon as they
are accepted; you can have the latest information about publications in your
field.
10. Present papers at conferences before submission
o Present your papers at regional, national, or international conferences. You
may get surprisingly valuable feedback.
o This also is an important way for you to become familiar with others working
in the same area.
o Presenting papers within one's department is not effective. Except in top
schools, most of the faculty in a typical department with 20 or fewer members
are not familiar with the subject, and with due respect to their expertise,
they generally are not qualified to make substantive comments on your topic.
11. Do not distribute unpublished papers to strangers (at big conferences)
o If you do, your desire to become well-known may be temporarily gratified,
but the penalty can be harsh later.
o Some people might steal your idea and submit a closely related paper sooner
than you do.
o You get no credit.
o Distributing papers is okay in a closed circle of researchers, where
everybody knows each other.
12. Only the tough get going
o One gets rejection letters more often than not. This is inevitable!
o Develop a thick skin and be a good loser. This game is not for the
faint-hearted. If you cannot swallow rejection easily, don't submit papers.
o A good paper deserves at least three chances at publication in ranking
journals.
o If you ignore a rejected paper more than one month, you are likely to lose
interest. Do something about it.
o Bad luck eventually comes to an end.
13. Get to know one hundred people active in your field
o There are about a hundred people in your field who are likely to be referees
of your papers.
o Prepare a list of one hundred active people in your main research areas. Try
to meet them within a five-year period.
o Present papers at, or at least attend, two professional meetings a year.
o When presenting papers or attending regional, national, or international
meetings, try to get to know these people. How? (Think!) This is your best
opportunity for networking.
14. Maintain contacts
o Maintain contacts with other economists via telephone, fax, or e-mail. Do
not send copies of your papers to them unless requested to do so.
o What to do when they don't respond? Think!
o You also need these contacts later: they can write letters of recommendation
when you seek promotion and tenure.
________________________________________
Articles and Books
15. A journal article is preferable to a book
o Don't publish a book, at least not before getting tenure.
o Readers find it easy to remember if your papers were published in journals
because they are often abbreviated like AER, JPE, RIE, etc. They might even
remember the years of publication.
o They won't remember your books, unless the titles are extremely short and
sexy.
Life of a publication
o The life of a book is about 1 to 2 years.
o The life of a journal article is about 10 years.
o Publishers will not spend much money to advertise your book because profit
margins are small.
o Accordingly, most economists do not know whether you published a book, let
alone know the title.
o Bragging to your colleagues about your recent book is like introducing
yourself by long names with 10 or more words.
o Authors who have published an article in the same journal feel friendly
toward you. It creates a bond among the authors.
o Book authors operate alone.
o Researchers know that books do not go through the refereeing process.
Weight of a publication
o Your department or division may not clearly specify quantified weights to
evaluate your research.
o But rest assured that they are there; a given number of papers in certain
journals or certain ranks, etc. These standards are developed by consensus,
and you can find these standards by checking the records of those who received
tenure recently.
o Journal rankings often are used to evaluate the quality of your research.
o All things considered, the following weights could be used:
 1 = an article in a good journal
 0.5 - 1 = a whole book, maybe 2 if it is very popular.
 0.1 = a chapter in a book someone else edited.
o Textbooks do not count.
o Handbooks and some special series might be treated like a journal because of
their long shelf life (10 + years).
o Do not give away your precious paper as a chapter of a regular book, unless
it appeals to your altruistic desire to help others.
16. A journal article first
o First, publish your original idea in an article.
o Then maybe in a book, not vice versa.
o Journals will not knowingly publish an article if the substance was
published in a book previously.
________________________________________
Collaboration
________________________________________
17. Cultivate coauthors
o Find seasoned coauthors with publication experience and share the glory.
o Working with your advisors is a good idea, at least for the first few years
after receiving a Ph.D.
o You have to become independent at some point, though.
o Acting alone is a risky strategy, especially for those just out of graduate
school.
o With seasoned coauthors, the probability of acceptance will likely more than
double.
o Through your coauthors, you may be introduced to an established group of
economists.
o You also may learn how to write better papers.
Weight of coauthored articles
o Whatever rankings are used, given the quality, the following weights may be
used more or less as a guide to estimate the overall impact of joint articles:

 1 = an article (sole author).
 0.75 = first author in a joint paper.
 0.7 = second author in a joint paper.
 0.5 = an author in a paper with three authors.
 1/n = four or more authors. (Don't do this, except in certain fields
[e.g., agricultural economics], where it is more acceptable. You will be
included in "et al.")
18. Make an agreement with coauthors ex ante
o It is best to divide up the work with coauthors ex ante. This minimizes the
chance of free riding when the paper is complete or accepted.
o Be considerate when determining the order of authors.
o To assure a long-term relationship, alternate the order of appearance,
especially when the contributions are roughly equal.
o If you insist on alphabetical order just because your name precedes the
others, they may not come back to you for further collaboration.
o Another practical idea: flip a coin.
19. Maintain collaboration
o If a personality conflict develops, collaboration does not work.
o It takes time and effort to cultivate relationship with coworkers. If you
have found a good working relationship, don't tamper with it to obtain small
gains.
o If you do seek small gains, it is difficult to restore a good relationship
should you change your mind later.
20. Be patient with inactive coauthors
o Be tolerant of your coauthors.
o Remember that the sum of subjective contributions of coauthors of a paper
always exceeds 100%.
o Removing an inactive coauthor from the paper may not give you peace of mind,
especially if it is done insensitively.
o Keep pace with your coauthors. If a coauthor does not contribute anything,
caution must be exercised. Often the animosity generated is not worth the
gain.
o If a joint work is being terminated because of unforeseen developments, make
it clear who holds the ownership of the disputed papers. This eliminates
untold misery later.
________________________________________
Choosing Topics
21. Do not waste time on dead or dying topics
o If your most recent references in a projected paper are ten years old, it
will be difficult to publish it. It is a dead issue. Do not start such a paper
(until you get tenure)!
o If the most recent references closely related to your paper are 5 years old,
it is a dying issue. Editors are reluctant to accept such papers, even if the
referees recommend publication.
o It is difficult for the editor to find suitable referees for outdated
topics.
o Your inability to find sufficient references indicates
 You have not read the literature.
 Others are not interested in the topic, hence, it is unlikely to get
published.
 No problem! Dig further.
o If the work is completed already, cite some papers that are more recent.
22. Do not write papers with breakthrough ideas at first
o Avoid writing about your breakthrough ideas, at least in the early stage of
your career, unless your mentor is the editor of a major journal.
o Papers with breakthrough ideas are not often published.
o Wait until you get tenure to tackle breakthrough ideas.
o "I told my own young colleagues that they should preferably start off with
the received wisdom with some changes until they get their tenure." -Douglas
North, 1993 Nobel Laureate in Economic Science (see Nyaw and Yu, 1995).
o If you do advance breakthrough ideas your papers will be rejected, and they
might reappear in a modified, clearly written paper by someone else later.
o After you are established, perhaps you can tackle breakthrough ideas, and
become better known, instead of publishing many papers with minor ideas.
o Or as you gain more experience, you may find that the ideas turn out to be
trivial.
23. Extend existing literature
o The bulk of papers published today are modifications of the existing
literature or tests of existing theories.
o Something in the paper must be original.
o Duplication is not an extension of knowledge.
24. Write something creative
o A journal's primary goal is to publish original ideas.
o A good journal is interested in disseminating new ideas, not in publishing
papers that elaborate some existing ideas or examine the implications of a
minor change in assumptions.
o These papers only show that some results do not necessarily hold. Such
efforts are basically a comment on someone else's paper.
25. Mix ingredients of other papers
How does one extend the literature? Suppose there are two important papers in
the literature,
p1 = {A, B, C, and D}, p2 = {C, D, and E}
where A, B, ... are ingredients.
Let pnew = {A, B, E} be a new paper.
o Does the new combination make sense? Does it describe an important economic
phenomenon in a certain country or does it capture an interesting situation?
o If pnew = {A, C, X} where X is totally new, and if it makes sense, it may be
an original idea.
o Original papers add something new and dare to eliminate some old notions. Do
not worry about compatibility with old papers.
26. Write on interesting subjects
o There must be an interesting story, a story that nonexperts?who would skip
all the equations?would find intriguing.
o Equations should not dominate the paper. People lose interest.
o Controversies and debates stimulate reader interest.
o Before writing, answer the question: what new ideas or results does this
paper offer?
o You have to demonstrate that there is some interest in the topic on which
you are working.
________________________________________
Comments or Notes
27. Avoid writing comments on other papers
o Writing comments is risky because you are at the mercy of the original
author.
o If a comment or note is rejected, you cannot send it anywhere without
substantial rewriting; it is too short.
o When a comment or note is rejected, abandon the note or expand it to a
full-blown paper.
o If you add something new while making the original author shine, you might
succeed. For instance, if you name the result after the original author, it
makes everybody happy.
o If you point out errors in the original paper, your referee (the original
author) will find something wrong in your comments also, whether they are
real, imaginary, or spurious.
o Occasionally, writing comments is okay (once every few years). But do it
quickly, while the editor's memory is still fresh.
o A safer approach is to write an independent paper.
o AER has a standing policy not publishing comments, even to correct errors.
Remember Robert Fulghum’s advice “Clean up your mess”?
o Do not develop a habit of writing comments on others' work.
28. Do not correct small errors others make
o It is dangerous. This practice rarely earns you respect.
o You may not be right. As you rush to prove your points, you may not have
grasped all the fine points of the original paper.
o Even if you are right, the original author may lurk in the trenches where
he/she can stage a counterattack and damage your credibility in the future.
o You also don’t like to have your errors pointed out.
o Why beholdest thou the mote that is in thy brother's eye, but considerest
not the beam that is in thine own eye? (Matthew 7:3)
o The referee may then be negative toward all your future papers.
o Communicate with the original author before you submit your comment. If you
are diplomatic and fortunate, you might acquire a friend who would look at
your papers with favor in the future. On the other hand, you may find an enemy
who will always find fault with your future papers.
o
Writing Strategies
Cover Page and Cover Letter
1. Prepare a perfect cover page and an abstract
o The cover page should contain complete correspondence information about the
submitting author:
 postal address
 telephone and fax numbers
 e-mail address
o If you move, give your new address to the editorial office.
o If updating a paper, give the current date (or month and year).
o Do not mention when a paper was first written and when it was revised. The
editor can tell how often the paper has been rejected, and may erroneously
conclude that it should receive the same treatment. If you really need the
information for yourself, you can add such things as a non-printing comment.
It is probably more convenient to maintain a separate record that shows the
status of all your unpublished papers.
o If the referee figures out that the paper has been rejected more than once,
he/she is more likely to recommend rejection.
o The abstract and the paper should be prepared together.
o When the paper is finally accepted, the abstract has to be written, but your
memory is hazy. It is better to do it when your memory is fresh.
o The abstract should appear on the second page. Then if the editor rips off
the cover page, the abstract will still reach the referee.
o Eliminate typographical errors in the cover page and the abstract. This is
an absolute minimum courtesy. If there is an error, it is a sign of gross
neglect.
o Of course, you have to check the spelling for the entire paper, and you
should do that every time you revise the paper.
2. Don't explain how important the paper is in the cover letter
o Editors do not read it.
o Maybe the secretaries do.
o This is a signal that you lack experience and that you are not confident.
o One or two explanatory sentences may not hurt. (You may pass the initial
screening.)
________________________________________
Introduction
3. Devote half the writing time to the introduction and conclusion
o Once the ideas of a publishable paper are roughly formulated, writing should
be done within a month. Otherwise, you lose interest. You may even forget
about the entire paper.
o About half of your writing time should be devoted to writing the main body
of the paper, which should be done first.
o The remainder of your effort should be devoted to writing the introduction
and conclusion.
4. Get their attention early
o Provide evidence of why it is interesting (i.e., why it should be published)
in the introduction.
o If an apple does not taste good at the first bite, one simply throws it away
without giving any thought on the nutritional value hidden in the apple.
o Likewise, most referees make up their mind at the first bite, i.e., within
15 minutes of reading a paper.
o If the referees don't like a paper, they begin to look for reasons to
justify why the paper should be rejected.
o If the referee loses interest from reading the introduction, he/she might
postpone reading the paper.
o If a paper is set aside, it could be several months later when the referee
picks up the paper again, probably if and when he/she receives a reminder
about the review. This is one of the major reasons why it takes a long time to
get a report.
o Do not repeat the concluding remarks in the introduction.
5. The introduction should be two pages or less
o If the introduction is more than two pages, it is too long.
o Shorten it to 2 pages or 1/6 of the paper, whichever is less.
o If you write more than two pages, then either
 you are discoursing a lot about other people, in which case you are
sending a signal that your contribution is minor, relative to the literature,
or
 you are discussing too many technical details, which do not belong in
the introduction.
6. Discuss real world examples
o Pass the relevance test by providing citations, statistics, or anecdotes of
real world examples.
o Then the referee cannot say the paper is uninteresting, the most common
reason for rejection.
o If the referee says it is not interesting, it is a value judgment and there
is no appeal! No editors will publish an uninteresting paper.
o One important purpose of the introduction is to prevent the referees from
making that disparaging remark.
o Without this sound footing in the real world, your paper may give the
impression to readers that it provides a profound solution to nonexistent
problems.
7. Imitate skillful writers
o Observe how other successful writers introduce their topic, cite literature,
and get on with their task.
o Imitate their words and phrases, and modify them to suit your purpose.
o It is easier to imitate what someone else has written than to create a
totally new paragraph.
8. Do not plagiarize
o The word “plagiarize” means to “steal and pass off as one’s own (the
ideas or words of another).” (Webster’s Third International Dictionary,
1986)
o Remember Robert Fulghum’s advice “Don’t take things that aren’t yours.”

o If you do, you will pay dearly later when your work is published. You are
lucky if the paper is not published!
o If you are quoting statements made by another writer, use identifying
quotation marks.
o Some people suggest that one should not copy more than three consecutive
words without identifying quotation marks. This is extreme advice that no one
can follow.
o Do not copy, but summarize the contributions of other writers in your own
words to the extent that they are related to the subject of your paper.
o Mention the cited author with year of publication in the text and give the
exact source in the reference section.
9. Do not use I
o Some authors do get away with I.
o Referees are generally biased against egocentric persons.
o Take the writing task seriously, not yourself.
o "The paper achieves...." sounds softer and more humble than "I did this."
o Avoid starting a paragraph with I.
10. Create a packet of related articles for each paper
o All cited and other related papers must be at hand.
o This practice saves time, especially when writing the introduction and
conclusion, and when you revise the paper.
o If you maintain the background packet, you do not have to go to the library
every time you revise the paper.
11. Treat others generously
o Emphasize the importance of the paper being written, but not at the expense
of others. They are probably your referees and they are sensitive.
o Don’t hit people (Robert Fulghum). Do not hurt their feelings.
o When mentioning the works of other persons, avoid using negative terms.
o Examples:
 "The deficiency of Smith's approach is..."
 "The problems of these papers..."
o Papers that attack others are likely to be rejected, especially when the
authors or their friends become your referees.
12. Avoid predominantly citing your own works
o The referees may think you are a self-centered clod. There are others who
have contributed to the literature.
o If the first page only mentions your past work, and not that of others, it
means either
 you are probably digging into an area in which no one else is
interested?this implication is bad?or
 you are an egotist who disregards the contributions of others, which
is even worse.
13. Cite the papers of potential referees in the introduction
o In many situations, whether your paper is accepted or not primarily depends
on who referees it.
o If you offend the referee by your thoughtless comments, this paper and many
of your future papers will have no place to go.
o Important references should be mentioned in the first page.
o Hopefully, the editor will read the first page (or the next) when choosing
the referees.
o The editor may choose referees from those mentioned in the introduction and
references.
o Works of potential referees should be mentioned in the introduction, rather
than buried deep in footnotes or the main body.
14. Give (accurate) credit generously to the most likely referees
o Be generous to all authors cited, but particularly to those who are likely
to be referees.
o Explain why their works are significant for your analysis.
o Write one or two sentences about the contributions of each of the most
likely referees and how their works are related to yours.
o This takes up less than 1% of the space, but it can affect the probability
of acceptance significantly.
15. Find quotations from well-known authors
o This strategy increases the credibility of the paper.
o For instance, if John Maynard Keynes or Kenneth Arrow said something about
the topic, it is difficult for the referee to argue that your paper is
uninteresting.
o Quoting a live, famous person is more effective; his or her students might
be referees.
o Do not quote dead people too often; they won't be your referees. (No pun
intended.)
o Do not quote yourself. This implies narcissism or lack of exposure to the
thinking of other economists.
16. Do not be apologetic
o You may acknowledge the limitations of the approach only once in the
conclusion.
o But do not apologize for what the paper cannot do.
o The more you mention to the referees what the paper does not do, the less
contribution it seems to make to the literature.
Preparing the Main Body

17. Prepare a rough outline before writing
o Sketch briefly the content of each section. Then generate the text. Smooth
out the connections. Without this rough blueprint, the paper often evolves in
a different direction than you intended.
o This blueprint reduces the chances that you will lose direction and dwell
too much upon minor points.
o This sketch needs to be changed as you go.
18. Start writing before the paper is finished in your head
o The precise connection of words from beginning to end cannot be done in your
head, except by a few geniuses like Shakespeare.
o A 15-page paper may contain about 4 - 5,000 words. Writing a paper is like
stringing pearls to make a necklace. There is an optimum order for these
pearls to form a paper, and some pearls are better left out.
o Begin the main body of the paper with empirical or theoretical results. Then
create the introduction and conclusion.
o Tables and references may be added as needed.
19. Do not read too much
o Do not read too much before you begin to write. It can interfere with your
own thinking and writing.
o Imagine how much time a prolific writer would spend reading the
contributions of other people.
o It is impossible to read every paper ever written on a subject.
o Remember your goal is to write and publish a paper, not to read everything.
o You have other important things to do (e.g., taking care of spouse and
children)!
o If your family is neglected, what good is your paper?
o If you read a dozen papers on a topic, you should have enough material to
write a paper. Now add your own ideas to this base of knowledge.
20. Develop consistent and simple notations
o Invest enough time to design efficient notations for your papers.
o Do this not just for one paper, but for most of your papers. This helps you
remember when you revise a paper.
o If the notations are confusing, the paper cannot be very illuminating.
o Each paper may have some notations that are specifically tailored for the
task. But the variables should come from a well-designed and consistent set of
notations so that you may readily remember what they stand for.
21. Strike a balance between theory and applications
o A theoretical paper should say something about policies, applications, or
empirical work.
o An empirical paper should say something about the theory that led to the
empirical work.
o Check the preferences of the journals that you are considering.
22. Divide long paragraphs
o If there are two or more ideas in a single paragraph, split them up.
o Break up long paragraphs even if they contain a single idea.
o Readers tend to skip long paragraphs. They discourage referees and readers
from reading the paper.
o The eyes of readers are subconsciously looking for open space. This is why
important equations should be displayed, rather than buried in the text.
o No paragraph should be longer than half a page.
o As a general rule, a paragraph should have more than two sentences.
23. Each full page should have more than two paragraphs
o A paragraph extending over a page indicates that you are not an experienced
writer.
o Referees and readers skip long paragraphs.
o When there are many equations, it is easy to forget to control the length of
a paragraph.
24. Summarize theoretical findings in propositions
o If you do not want the referees to miss important results, repeat them in
propositions.
o The referees do not read every word you write. They are more likely to read
the displayed items.
o Minimize the number of words in a given proposition.
25. Use tables to summarize results or to compare with the literature
o Tables provide another way to catch the attention of referees.
o Avoid too many numbers in one table.
o Do not present more than three tables, except in empirically oriented
papers.
o Do not present more than six tables even in empirical papers.
26. Minimize numbered equations
o There should be some equations. Otherwise, the referees might think that it
is a purely descriptive paper.
o But do not include too many equations. A paper with more than 30 equations
seems difficult to read.
o Do not display every equation. Less important equations can be buried in the
text.
o Not all equations need to be numbered.
o Use primes or other variations such as (3') or (7a), (7b), etc. to group
related equations.
o If there are more than a score of equations, move long derivations to the
Appendix.
27. Simplify figures
o A (good) figure is worth a thousand words.
o Do not use too many curves, lines, or labels.
o Ten years after publication, readers may not remember anything about a
paper, not equations nor derivations. But they may recall a figure.
o As a general rule, a paper should not contain more than two figures and
rarely more than three.
o Too many figures suggest that the paper represents a low-tech research
effort.
________________________________________
Conclusion
28. Summarize the contribution briefly in the conclusion
o A paper needs a concluding remark. A note does not, but it may include such
a remark.
o Mention the limitations of the results (without being negative).
o Discuss how the theory may be extended in certain areas.
o The referees may be interested in writing a related paper. If they are
honest, they would need your paper as a basis, and hence are likely to
recommend acceptance. That?stimulating a reader to extend your research?is
your contribution.
o Compare your results to those in the current literature.
o If the literature does not have comparable results, discuss how your paper
is related to the literature.
o Do not repeat some portion of the introduction in the conclusion.
29. Discuss policy implications
o Explain how the theory applies to real world examples.
o Example: In practice, A is used, but you recommend B, etc.
o Do not rehash what you already said in the main body of the paper.
Especially, do not copy and paste it in the conclusion.
o If you do, the referees will know you are not articulate.
o Present the bottom line. Mention the implications for policy makers,
practitioners, or other researchers.
________________________________________
Abstract and Title
30. Write a provokative abstract
o Write the abstract only after the conclusion is written.
o The referees read it more often than any other paragraph in the paper.
o In 15 seconds, you have to convince the referees (and readers) that they
should proceed with the rest of the paper.
o So do an excellent job here.
o If it is boring, your paper is hopeless.
31. Choose an interesting title
o Give the paper an eye-catching title.
o If the title is boring, readers will avoid your paper even when it is
published. The paper won't generate many citations.
o Never try to squeeze the content of the paper in the title.
o Giving a title to a paper is like naming your child. The title should be
short.
o One line is best. Never use more than two lines.
o Avoid "On the...". It implies that the paper is actually a note. Because it
is on a well-known subject, the editors are led to believe that the paper
probably contains little that is new.
________________________________________
References
32. Minimize references
o An inexperienced writer rarely resists the temptation to cite all papers
that have ever been written on the subject.
o This practice may be appropriate for a doctoral dissertation, but not for a
journal paper.
o An ideal number of references is one dozen. A practical upper limit is
twenty.
o For all papers, follow the reference style of a well-known journal in the
field.
o Do not revise the reference style each time you submit the paper. The
acceptance decision is not based on the style of your references.
o After the paper is accepted, you can use the style of the journal in
question.
33. Include references to authors who are known to like your papers
o Perhaps they might become referees.
o Include references to people with whom you have had favorable
correspondence.
o This is not to bias opinions, but to get a fair hearing.
o Referees have to make a conscious effort and must be alert in order to be
fair to unknown authors.
o Include liberal references to famous economists, dead or alive, who are
unlikely to be your referees.
34. Delete or hide the references to undesirable potential referees
o Even with double blind reviews, one can often guess the identity of the
referee from the report because of references and writing style, etc.
o Editors often select referees from your references.
o If some referees consistently recommend rejection of your papers, drop their
papers from your references (in the initial submission).
o You can add them later (after the paper is accepted).
o This may require rewriting the introduction with a somewhat different
perspective, but it is probably worth the effort.
o Depending on the journal, you may ask the editor to eliminate some persons
from the pool of referees. But you should ask informally (e.g., via e-mail) in
advance if it is okay.
35. Cite your own articles
o An article is considered "important" if it is cited 30 times or more by
others.
o Cite your own related papers, provided that they were published or are
forthcoming in a prestigious journal. Others may look up your other papers and
cite them.
o But do not cite too many.
o If you have a good reputation, this practice can be useful because the
referee may figure out that it is your paper.
o Do not cite your own unpublished papers or publications in an obscure
journal. The editors and referees may conclude that the current paper also
should be published in such journals.
o Do not cite your dissertation. The referees will know you are inexperienced.

o Do not cite someone else's dissertation. The referees may erroneously
conclude that you are him or her or a close associate, all of whom are
inexperienced.
________________________________________
Endnotes and Appendix
36. Put technical, detailed comments in notes
o Combined endnotes, tables, references, and appendix or appendices should be
smaller than the main body of the paper. Otherwise, readers wonder “where is
the beef?”
o The main text should be free from technical details, and the major ideas
should emerge from reading it.
o Intellectual clutter should be relegated to closets, i.e., notes.
o Use notes to insert references and to make points that do not distract
typical readers.
o No more than 10 endnotes should be provided. Avoid them like the plague
(Horowitz, 1995).
o Notes should be short, not exceeding a page and never more than two pages.
37. Put long derivations in an extended note or an appendix.
o Long derivations of an essential result or an equation which may be over
half a page can be included in an extended footnote, if there is risk of
boring readers.
o If there are two or more extended notes, they should be converted to an
appendix.
o If the derivation is purely mathematical without apparent insight, it should
be in an appendix.
38. Notes intended for referees should not be in the appendix.
o Anything intended for referees' eyes only should be explained in the notes.
o Do not detach such notes from the paper, but write "Not for Publication" on
them. If you detach the notes from the paper, they may not reach the referees.

39. Your paper should not exceed 25 pages
o If this is difficult, at least keep the text within 20 pages (Horowitz,
1995). This is the amount the referees would read.
o As the length of the paper increases, the probability of acceptance
decreases. The referees are more likely to find something wrong.
o As the length of the paper increases,
 You are more likely to make mathematical errors.
 The chance that the referee thinks you made a mistake increases (even
when you are right).
 You are more likely to make statements that will offend referees.
Preparation and Submission
Preparation for Submission
1. Sit on the finished version for one week
o After the paper is completed, do not immediately submit it to a journal. (It
is not finished yet.)
o You invariably will find many small errors in text, notations, explanations,
or missing references, etc. in your finished paper.
2. Reread the introduction, conclusion, and abstract before submission
o Reread these three parts carefully before you submit the paper to a journal
and eliminate all typographical errors and other embarrassing mistakes.
o A typographical error on the first page of introduction or abstract
indicates that the author is careless.
o Such errors tend to lead referees and editors, rightly or wrongly, to
conclude that the paper should be rejected. They conclude that the author is
likely to be sloppy in substance as well. And they might be right.
o If you don't proofread your own introduction, why expect the referees to
spot and correct all the errors?
3. Use, but do not rely totally on spelling checkers
o One should always check spelling before submission. But there are no
substitutes for reading the papers personally.
o Spelling checkers do not check word meanings.
4. Do not arouse envy
o Do not use fancy fonts or expensive bond paper.
o Do not cite too many of your own papers.
 The referees might feel that you have published too many papers.
 The referees might feel justified to recommend rejection of your
paper.
 Especially when he/she received one recently.
o Do not thank famous people in the acknowledgment, at least not in the first
submission. The referee's contacts may not be as good as yours.
o Do not thank family members. This is understandable, but it is
unprofessional.
5. Use common sense
o It is not a good idea to send a hand-written submission letter. The
submission letter contains critical information about the author (address,
telephone number, e-mail address, etc.) Your scribbling may be a challenge to
the deciphering ability of the editors or their assistants. A small
typographical error in the address might make a letter to the author
undeliberable. Here is an example:

o Use a sturdy envelope, especially, if you are sending a manuscript to a
foreign country. An enclosed check might be missing from the package by the
time it reaches the editorial office.
o You do not want your package to arrive at the editorial office looking like
these:



6. Consider electronic submission if allowed by journals
o Journal offices increasingly are more willing to receive electronic
submissions.
o Electronic submissions are faster and safer.
o Word processor files can damage the hard disk of the journal office. For
this reasons, they prefer PDF files.
o When submitting to journals that adopt double-blind refereeing process,
submit the cover page and the main body separately. Remove your name in the
document property (Your computer may record it automatically.)
o Experienced people report that Acrobat PDF Writer does not always produce
dependable PDF files.
o Use the dependable Acrobat Distiller. For instance, after the Acrobat is
installed, you can print a Word document using Acrobat Distiller and save it
at a desired drive. You can then e-mail the file.
o After a PDF file is produced, go over it to see if all symbols are properly
represented. If a symbol is not properly converted by Adobe Distiller, try
retyping it using another font. Avoid using nonstandard symbols, because
Acrobat Distiller may not convert them properly.
o Visit the NSF site concerning problems you encounter when creating PDF
files, http://www.fastlane.nsf.gov/a1/pdfcreat.htm.
________________________________________
Working Papers
7. Present an early version as a working paper
o If a paper contains enough substance of a roughly sketched idea, you may
offer it as a working paper, just for the record.
o Distribute it to a dozen trusted friends in your field to get feedback.
o But do not distribute it widely.
o Working papers can attract coauthors, and a revised version may be published
later. When you are up for promotion and tenure, the working papers provide
evidence that you have started the work.
8. Do not submit your working paper to an electronic journal
o Get ready for the future of publishing. Most journals will become available
electronically over the coming years. Hard copies may still be available, but
they will be expensive because of limited print runs.
o You may submit abstracts to journals on the Internet, but it is not
advisable to post the actual articles.
o For legal purposes, the electronic publications may be treated as
publications. But for tenure and promotion purposes, they do not count as
publications. This is a problem.
o It is easy for someone to manipulate the electronic copy (even PDF or PS
files), modify it a little, and submit it to another journal under a different
title.
________________________________________
Acknowledgment
9. Remove negative clues from acknowledgment
o In the acknowledgment, remove any reference to when the paper was conceived
or written.
o Editors of journals that adopt the double-blind review procedure are not
likely to send papers to persons mentioned in the acknowledgment.
o Do not thank in the acknowledgment the people whom you would like to serve
as referees. Acknowledge them after the paper is accepted. Otherwise, they are
likely to be left out of the review process.
o Once you receive an invitation to publish, include an acknowledgment to the
referees, whether anonymous or not.
________________________________________
Submission
10. Eliminate any trace of prior rejections
o Do not indicate when the paper was first written. If the original version
was written a few years earlier, the editors and the referees clearly see that
it has been rejected a few times.
o Do not indicate how often the paper has been revised. This suggests you do
not listen and properly modify the paper to make it more publishable.
o In the references, eliminate any references to papers that were
"forthcoming" a few years back. This not only indicates that your paper was
previously rejected a few times, but also that you are sloppy in updating the
references.
11. Submit your paper to a rising journal
o Good specialty journals are rising.
o The acceptance rate may be higher. Payoff is greater later.
o Identify and avoid the declining journals whose acceptance rate is low with
a diminishing payoff later.
o General journals, except for a few at the top, are expected to decline
because of increased specialization and the resulting drop in demand for them.
In general journals, "readers are confronted with a decreasing probability of
finding at least one important article" (Holub, Tappeiner, and Eberharter,
1991) in their field.
o In the 1970s, the top ten journals were general journals.
o In the 1990s, half of the top ten journals were field journals.
o As you become more specialized, an increasingly smaller fraction of papers
in general journals become relevant to your research. Accordingly, demand for
general journals is likely to decline.
o Increased specialization is more likely in the future.
12. Keep a log of research papers
o In the first two or three years when the number of articles under review is
small, it is easy to remember the status of your papers. Later, as the number
of articles increases, a log will prove invaluable.
o The purpose of a log is to
 know when to send a reminder to the editor,
 prevent resubmission of a rejected paper to the same journal, unless
of course, it is your intention to resubmit the rejected paper to the same
journal (after a change of editors), and
 avoid multiple submission of several papers to the same journal
within a short period of time.
o For each paper, note the pool of potential journals.
o When a paper is rejected, do not lose time resubmitting the paper to another
journal.
o Keep a log of the life history of each paper.
13. Do not submit two papers to the same journal in two months
o Especially if the two articles are related.
o Other things being equal, editors prefer to publish two articles by
different authors, rather than two articles by the same author.
o You may submit more papers to the same journal simultaneously if there is
more than one editor. They do not often communicate with one another. In this
instance, acceptance of one article by one editor does not adversely affect
the chance of another being accepted by a different editor.
14. Check for related articles in the journal being considered
o Try to find some related articles in the journal to which you wish to submit
your paper.
o Authors who published a paper on a related subject are likely to be
referees. The editor's memory is still fresh.
o Obviously, you need to say something about, or at least cite, their papers.
o Even if they are marginally related, try to incorporate their references.
Make some effort to explain how your work is related.
15. Avoid the journals which consistently reject your papers
o Haven't you learned your lesson yet?
o Avoid (temporarily) the journals which have rejected your papers
consistently, say three times in a row.
o The editor still remembers all those bad remarks about your papers.
o Wait until a new editor is appointed.
o First and middle names, as well as last name, often reveal the sex, race, or
nationality of the authors.
o If you have reason to believe that you are being discriminated against on
the basis of sex, race, or nationality, you may consider using initials
instead of spelling out the first and middle names.
o You may reveal your full name after the paper is accepted.
16. Use professional editorial assistance
o Particularly if you are not a native English speaker
o Editors will not publish papers with grammatical errors.
o It is safe to assume that referees are biased; they have an excuse to
recommend rejection when grammatical errors are detected.
o You can easily find a copy editor who charges a reasonable fee.
o Editorial help is available in the English department of any university in
the United States or the United Kingdom. If you live elsewhere, you need to
invest some time to develop friends located there. You may be able to check
and expedite the editing process through them.
17. Know the preferences or biases of journals
o If a journal rarely publishes empirical papers, do not send one there.
o Similarly, if a journal rarely publishes theory papers, do not submit one
there.
o If you suspect discrimination, check the past issues of the journal in
question. This will reveal surprising insights.
o Preferences are known; biases are difficult to detect.
o There are three types of journals:
 Association journals (AER, Econometrica, etc.)
 University journals, managed and edited by university faculty (QJE,
JPE, etc.)
 Journals published by commercial publishers (Blackwell,
North-Holland, etc.)
Problems of Journals
o Association journals: Editors change every few years, and they tend to
accept more papers by colleagues and friends while they are at the helm. Since
the editors are chosen from among a few major institutions, they tend to get a
larger share of publications than under ideal academic conditions. Subsidized
by associations.
o University journals: Promoting truth and knowledge is not necessarily the
primary concern of these journals. The universities need to protect their own
interests. They should set a good example by announcing that their editorial
standards are not compromised to protect their own interests, but do they have
the courage? Subsidized by universities.
o Commercial journals: To maximize profits they are least likely to have
preferences or biases. However, they cannot survive without reader
subscriptions.
Clan Power and Publication
o Roughly half of the papers published in some 40 high-ranked economic
journals are never cited by others (Holub, Tappeiner, and Eberharter, SEJ
1991). Journals included in their studies were: AER, CJE, EJ, EER, IER, JDE,
JEL, JET, JMCB, JPE, JPubE, OEP, QJE, RES, REStat, SEJ, Econometrica,
Economica, and Economic Inquiry among others.
o Even their referees would not cite these papers. This indicates that they
did not place a high value on the papers. Why would these referees then
recommend their publication?
o This finding suggests that in each field there may be small groups that
exert some influence by recommending publication of the papers by their clan
members.
o The clan members, implicit or explicit, are rent seekers. They recommend
publication of their own papers at the expense of nonmembers.
o An effective way for a newcomer to beat the clans is to join them by
collaborating with a clan member.
o The double-blind review process tends to reduce the power of clan members.
o Even with the double-blind review process, referees often know or guess the
identity of authors because papers are circulated prior to submission.
o Circulation of working papers prior to acceptance effectively reveals the
identity of the author and increases the rent that accrues to clan members.
How long to wait for results
18. Contact the editor after six months
o Editors do not have an alarm clock that goes off for each paper after a
certain period of time has elapsed.
o If it has been six months from the date of acknowledgment, you should
contact the editor.
o If you are counting from the date of your submission, allow seven months.
o Remember that the editors of many top journals are older and lack computer
skills. So e-mail is not an option. If this is the case, write a polite
letter.
o If you do not get a response within two months, send a second inquiry.
o Call the editorial office or inquire via fax.
o If you still get no reply after a third inquiry, you should not submit a
paper to such a journal again.
o An e-mail inquiry is okay, if the editorial office is so equipped.
o Note that e-mail inquiry is less formal and e-mail traffic is increasing.
o E-mail messages are less reliable; they may not reach the editorial office.
Rejection and Revision
Rejection
1. When rejected, try again
o Even Nobel Laureates get rejection letters.
o Papers lying dormant in the file drawer do not bring any good news!
o Submit the paper to another journal within one month. But wait!
o If a referee points out a major problem, you need to address it.
o You do not have to revise a paper every time it is rejected.
o But if a paper is rejected 4 times, there is a serious flaw in the paper.
Find and fix the problem.
o Make a modest effort to incorporate the valuable suggestions of the referee
before submitting to another journal.
o Why? The same referee might get it again.
o Do whatever possible to make sure the negative referee does not get the
paper again. You are entitled to new referee reports.
2. If a "stupid" referee misunderstood your paper, it is your fault
o Truth hurts sometimes, but listen anyway.
o Some referees spend as little as 15 minutes reading your paper. Your paper
should be clearly presented, and it should be comprehensible by such referees.

o The typical referee spends two hours or more on your paper. Moreover, he/she
is an expert in the field. Find out why such an expert has trouble
understanding your paper and correct the problems.
o This "stupid" referee problem will not disappear until you correct it.
o There must be something valuable in those reports. Salvage and incorporate
them freely in your paper. (And you do not have to thank them.) This is not
plagiarism.
3. Do not get angry
o Do not brood over ways to get even with the referees or the editor. Your
energy then would be devoted to a counterproductive and unhappy task.
o Writing a rebuttal letter to the editor rarely reverses the decision. The
referee has to defend it, even if it was a bad report.
o The editor already has a stack of such complaints. One more is not likely
to change the editor’s decision, albeit there are exceptions.
o When the referee successfully defends the report (in the eyes of the
editor), you lose any capital you may have accumulated.
o Write only if it is a simple matter.
o Instead of trying to prove that the referee is wrong on several points,
explain why you might deserve a second or third opinion.
o Example: argue that there is no mathematical error, contrary to the report.
________________________________________
Revision
o There might be a time limit for resubmission, usually six months to a year
from the date of the invitation letter.
o If you do not intend to revise and resubmit the paper for whatever reason,
let the editorial office know of your intention (via e-mail/fax).
o Remember that for all practical purposes this is probably your last chance
to revise the paper. The probability that you will succeed is about 50%,
depending on the journals. Sloppy, rough revisions will surely result in
rejection. The editorial office will not continue to provide mediation between
the referees and authors because there are other papers demanding attention.
o You received an invitation to revise the paper because it might contain a
publishable idea. However, papers will not be accepted unless they are
presentable and polished enough for publication.
4. Be optimistic and get excited
o Don't blow it. (If you do, you may wait three more years to get another
favorable letter.)
o Take the time to do a good job. The goal is to ensure acceptance, not to
minimize the effort.
o Do not save your effort. Go the extra mile. You have a chance (about 50%).
5. Write a detailed response to individual referees
o Take every comment of the referee seriously.
o In a note to be transmitted to the referee, first thank him or her.
o Number all relevant comments and respond to those (explain what you did in
the revised paper).
o Indicate that you are doing everything possible and more.
o If you cannot accommodate the demands, thank the referee for the suggestion,
but offer explanations why they are beyond the scope of the paper or why it is
not possible at the time.
6. Do not attack referees
o Generally, it is not a good idea to berate the reviewers. Don’t lash out at
the referees.
o Although they may not have a favorable opinion of your paper, they took the
time to read your paper!
o Do not say: "The referee's idea is bad, but mine is good."
o Better to say, the referee has an interesting notion, but the proposed idea
is also good, particularly in light of this or that fact.
o If the referee makes a valid point (you can almost always find conditions
under which the referee's points are valid), explain why, due to this or that
difficulty, you are not pursuing that course in the paper.
7. Resubmit the revised paper within three months
o Remember that this invitation is based on reports by some referees who had
good first impressions about your paper. Do not wait until that positive aura
vanishes.
o Do not resubmit the revised version in one month, even if you worked on it
full time.
o If you do, the editor may think that you have not devoted a sufficient
amount of time to the revision.
8. Write just one paragraph a day if you hate to revise
o The referees or editors have asked you to do an impossible or dreadful task.
Then just write one paragraph a day. You can do that!
o This works when you know you can do it, you should do it, but you cannot get
excited. The situation requires careful self-inducement.
o As you write a little bit at a time, before you know it, you get fired up.
9. Listen to what the editor says
o It is important to glean the true message from the editor's letter.
o Do not try to bargain with the editor (unless he/she starts it).
o Share the editor's letter and referee reports with experienced colleagues.
They may have surprisingly different interpretations.
________________________________________
Resubmission
When your revision is completed, you should send the following to the editor:
o copies of the paper (as many as requested)
o cover letter
o packet for each referee.
10. Check the Revised Paper
o The cover page should contain complete contact information about the author:
(i) address, (ii) telephone and fax numbers, and (iii) e-mail address. This
allows the editorial office to contact you quickly should the need arise. If
you anticipate moving, provide your forwarding postal and e-mail addresses.
o The cover page of the revised paper should include the current date (or
month and year) of revision; you do not want the office to send an old version
to the referees by mistake.
o If there were any complaints about the writing style, try to get some
editorial assistance. Remember that many papers are rejected because of
writing style problems.
o Eliminate typographical errors in the cover page and the abstract. This is
an absolute minimum courtesy.
o Last, but not least, make sure that there are no pages missing in any of the
copies.
Cover Letter
11. Explain succinctly how you revised the paper
o The purpose is to convince the editor that he or she should not send the
paper back to the referees.
o If the editor already indicated that he or she would send the paper back to
the referees, then your letter also should explain how well you followed the
suggestions of the referees.
Referee's Packet
12. Prepare a packet for each referee
o Regardless of whether the editorial office is well-managed or not, you
should prepare a packet for each referee. Each packet must include everything
a referee might possibly need. Specifically,
 A copy of the original (or latest) report. The referee might have
lost the file or might not remember even vaguely what he/she asked you to do.
A copy of the report not only helps the referee remember what he/she said
about your paper, but it also constrains the referee not to deviate too much
from the earlier report. The editorial office also has copies, but you want to
ensure acceptance even when the office is not well staffed.
 A copy of the revised version. Make sure you have responded to every
comment of the referee.
 A response to the referee's report. Do not forget to thank the
referee. Explain what you did or did not do in response to every comment.
 If the referee said something which you and the other referee did not
agree on, include a common response to the referees. This might calm down the
problem referee.
Being a Good Referee
General Guidelines
• You are performing a valuable service to the profession. It is worth
doing well. It also is good for your spirit when you have done something
worthwhile for society.
• As soon as you receive a manuscript, make sure it is something you are
qualified to judge. If you had agreed to review because of a misleading title
and you are not qualified to do the job, return the paper to the editor as
soon as possible.
• A referee report consists of two parts:
o a cover letter with the manuscript number/title and your opinion, and
o the report itself intended to be transmitted to the author(s).
• E-mail reports are acceptable to most journals. If the editorial
office is modern and the journal is well-managed, e-mail reports should be
preferred to reports by fax or snail (regular) mail, because snail mail often
unduly retards the editorial process and fax reports often are difficult to
read because of low resolution and small letters.
• Consider sending the report via e-mail or fax particularly when the
editor is on a different continent. International mail is generally less
reliable than its domestic counterpart.
• To expedite the refereeing process, you may fax your cover letter and
comments. Use high resolution mode, if possible. Just in case, also mail the
report.
• If regular mail is chosen, include two or three copies of the report.
Lost the manuscript?
• If you lose the manuscript, apologize and ask the editor to send you
another copy. Editors understand that referees who travel frequently lose
manuscripts occasionally.
• Do not wait six months to ask for a replacement copy or to tell you
never received the manuscript.
If you do not receive the manuscript
• If within four weeks (six weeks for international mail) you do not get
the manuscript you agreed to referee, contact the editor. The manuscript is
either lost or has not been sent out.
How Does One Become A Referee?
Here is a brief answer in response to this frequently asked
question. If you are well established, you will probably get a fair share of
articles to referee. If not, there are two ways to become a referee:
• Submit articles to journals. If you write an article on a given
subject, editors often assume you are an expert in that area. You might become
a referee for papers on similar topics.
• Write a letter to the editors. You can express your willingness to
serve as a referee in the areas of your choice. It is a good idea to enclose
your curriculum vitae.
________________________________________
1. Do it promptly
o Nothing is more appreciated by the editor and the authors than a prompt
referee report. The future career of the author depends on your timely
service.
o Do it in 4 to 6 weeks.
o Don't be too prompt! Otherwise, you may get too many requests.
o Prompt and sincere reports are your line of credit. You may need it when you
submit a paper to that journal.
o Hard copies are acceptable, but you may e-mail the report.
o If it would take you more than three months to complete the review, inform
the editor about the delay.
2. Be a fair and constructive referee
o Do not react even if the author attacks your previous contributions.
o Remember the days when you were a tadpole and the referees were gentle to
you.
o Focus on the merits, not on the immaturity of the writer. Science advances
because the next generation is immature and willing to experiment.
o If you are unfair or sloppy in a referee report, the authors may strike
back. The editor will remember the incident, even if the decision is not
reversed.
o If it is outside your area of expertise, promptly return the paper.
o If the topic is in your area, studying the paper carefully may lead you to
write another paper.
3. Do not plagiarize
o Make sure that you do not plagiarize and steal the ideas in the paper,
either consciously or subconsciously.
o For instance, examine the motive of a referee who says to himself: "Hm.... I
can do better than this author without making all these stupid mistakes. In
fact, I am going to do it."
o If you want to borrow some ideas from the paper, even if it is badly
written, make sure you recommend its publication and explain how to revise it.
If the author gave enough ideas to you to write a related paper, perhaps you
should recommend its publication. Ask the editor when the paper will be
published so you can cite it.
o It is unethical to recommend rejection of a paper which gives you creative
ideas to write another paper.
________________________________________
Cover Letter
You can reduce untold amounts of frustration you may impose upon authors and
help the profession immensely if your cover letter includes:
o the manuscript number (it takes extra time to locate the manuscript without
it).
o the title (in case there is an error in the manuscript number, this ensures
that the editorial office locates the manuscript).
o your postal address
o your permanent e-mail address
o your summary opinion
 A. Accept in present form or with slight changes.
 B. Accept for publication after minor revision, with a suggestion
about the length.
 C. Reconsider for publication after extensive revision.
 D. Reject, with suggestions for possible submission elsewhere.
o If you did not recommend one of the above, your letter is not well written.
4. Cover letter should be brief, not technical
o Explain the reasons why you recommend that the paper be accepted, rejected,
or revised.
o If you would like the editor to accept the paper, your recommendation must
be strong.
o If you consistently recommend rejection, then the editor recognizes you are
a stingy, overly critical person. Do not assume that the editor will not
reveal your identity to the authors. In the long run, there are no secrets.
o If you recommend acceptance of all papers, then the editor knows you are not
a discriminating referee.
________________________________________
Report
o Prepare your comments that include your reasons, suggestions, and concerns.
o Comment on the manuscript's originality, clarity, contribution to the
literature, and relevance to real world problems.
o Make suggestions about its length, organization, tables, and figures.
o The bottom line is this: If there is an important idea in the paper, make
constructive comments (e.g., how to streamline the arguments, what parts
should be cut) and help the authors publish the paper.
o If not, say so frankly. There is no point in beating about the bush. If the
paper is clearly below the journal standards, detailed comments are
unnecessary.
o If you e-mail your report, go to Document Property and delete your name.
Your computer may automatically record your name as the author of the report,
which may be accidentally transmitted to the author.
5. When you write a negative report, avoid citing your own papers
o Like animals, referees often leave their marks in their reports.
o If you vote against publication, do not cite your papers. Someday the author
will become a referee and return the "favor" in the next round.
o The paradox of refereeing is this: When you are a referee, you are the
expert. When the other person becomes a referee of your paper, he or she
becomes the expert. Circumstances can change.
o Do not say in the report whether the paper should be accepted or rejected.
This belongs in the cover letter.
o Be careful with your negative reports. Do not demoralize the authors.
o If you consistently recommend rejection of all papers in your area, people
will stop doing research in your area. Soon the topic becomes obsolete and so
do you.
o Moreover, soon the negative word gets around and people in the profession
might figure out who you are.
o If your published paper is relevant, you may cite it, but it should be done
without hinting at the identity of the referee. Do not cite your unpublished
papers.
6. Write more than one paragraph
o If you do not, you are not a sincere referee, whether you are famous or not.
You should have given the job to others who would devote more time and care to
the review.
o Remember the authors have spent several months to years to complete the
paper. They deserve more attention.
o Remember the Golden Rule: Do unto others as you would have them do unto you.

7. If there is a new important idea, help the author to publish it
o Your recommendation should be independent of whether the authors have cited
your papers or not.
o Do not use the report as an opportunity to force the author to cite your
paper if it is tangentially related. This is unethical.
o Divine beings don’t write papers (What would be the point?) All papers
written by mortals have problems. Your role is not in finding all the faults
in the paper.
o If the author can fix the problems with reasonable effort, do not
overemphasize the faults. Then recommend publication (in the letter).
8. Write something good, something bad
o Mortals cannot write “perfect” papers. Even the best paper has some
problems, and you can ask the author to make improvements.
o You also can say something nice about the worst paper. Remember you are
dealing with a person, and your report should not inordinately demoralize the
author.
o Remember the days when you were a tadpole before you write a nasty report.
o You can recommend rejection for good reasons and still be kind to the
author.
9. Reports should be based on the ideas in the paper
o The first paragraph should be a summary of the contribution. The editor is
not knowledgeable in all areas.
o Your evaluation should be based solely on the merit or ideas contained in
the paper,
o And not on who wrote the paper.
o Do not make comments demoralizing the author in the report. Thomas Edison’s
mother was reportedly told by his teacher that Thomas was “addled” and will
never amount to anything.
o If there is a writing problem, it should be noted.
o Remember that English is spoken by only 8% of the world population.
o A righteous referee shows no favoritism. There is no justification for
favoritism.
10. Avoid pointing out mathematical errors
o Unless you are absolutely sure.
o If you are wrong, the author will protest, and the second referee might
agree.
o If you lose credibility, your future papers also are suspect.
o Instead of saying the authors made a mistake, you can say you cannot obtain
the same result.
o But if you are certain, say so and explain why.
11. If it is hopeless, say so, and save the authors from further misery
o Don't try to be too nice in order to salvage an unpublishable idea.
o Being a good referee does not mean you try to help everybody publish in that
journal.
o Inherent capacities cannot be exceeded. Regardless of your suggestions, the
author cannot improve the quality of the paper more than 50%. Remember this
when you recommend revision.
o Positive recommendations should be based on the quality of an anticipated
revision.
Qustions and Answers
If you have comments or questions, please contact Kwan Choi at
kchoi@iastate.edu.
1. The corresponding editor resigned. What should I do with the revised paper?

o While I was preparing a revised version, I noticed that the co-editor who
oversaw my submission is no longer a co-editor of that journal. In this case,
how would this affect the status of my paper in the near future? Is it
possible to go through a new round with another editor and his/her own choice
of referees? (Jaejoon Woo, OECD)
o Usually the corresponding editor is responsible for editorial decisions for
papers he or she received for about a year. Submit the revised version to the
editor who made the initial editorial decision.
o The resigning editor still wants to relinquish his responsibility quickly.
The revised version should be submitted within a few months.
o Otherwise, the editorial decision may be delegated to a new editor who may
not be as favorable toward your paper as the first editor.
2. Professional technical editors are helpful
o I have found that a professional technical editor is also very helpful for
struggling beginning writers who have English as a first language (Bob
Coleman).
o Professional technical editors can make many useful suggestions. You can
accept some or all of their suggestions. It does not really matter whether
English is one's first language or not.
3. Where do I find a copy editor?
o Since I am not a native speaker, I want somebody to help me to check the
grammar and polish the paper. I understand that the service is not free, and I
am willing to pay. In fact, I am willing to pay a premium, if the work can be
finished before Thanksgiving. (Xioayan Zhang).
o Contact the English department of your institution to locate copy editors.
There should be many editors who can help you. They usually charge $10 - 20
per hour. Even graduate students in English department are very good.
Sometimes retired professors are willing to copyedit papers or theses.


Read more!